ORTHOPEDIC MCQS ONLINE QUESTION BANK H1E
ORTHOPEDIC MCQS ONLINE QUESTION BANK H1E
1411. (1764) Q4-2162:
All of the following are characteristic of hemangiomas except:
1) 70% of hemangiomas are visible by 4 weeks of age
3) Hemangiomas are three times more common in woman than men
2) 70% of hemangiomas regress by 7 years of age
5) None of the above
4) All cavernous hemangiomas regress by 12 years of age
Cavernous hemangiomas are noninvoluting hemangiomas and require surgical excision.Correct Answer: All cavernous hemangiomas regress by 12 years of age
1412. (1765) Q4-2163:
Pseudoaneurysms differ from true aneurysms in that:
1) Pseudoaneurysms are expansile.
3) Pseudoaneurysms have a fibrous wall.
2) Pseudoaneurysms occur secondary to trauma.
4) Pseudoaneurysms frequently occur in the ulnar artery.
Pseudoaneurysms occur secondary to penetrating trauma and have a fibrous wall, compared to true aneurysms that have all the elements of an arterial wall.Correct Answer: Pseudoaneurysms have a fibrous wall.
1413. (1766) Q4-2164:
Neurofibromas are characterized by all of the following except:
1) Café-au-lait spots
3) Dumbbell tumors
2) Axillary freckles
5) Characteristic oval avascular tumor in continuity with nerve trunk
4) Ease of surgical excision
Neurofibromas are benign tumors of neural origin that are transmitted as an autosomal dominant trait with variable penetrance. Neurofibromas are associated with cutaneous manifestations like cafe-au-lait spots and axillary freckles. They may be dumbbell shaped and can be identified with magnetic resonance imaging, which is especially helpful for deeper multiple lesions. They are infiltrative, making excision with preservation of peripheral nerve function difficult thus requiring bridge grafting for significant motor or sensory funtional requirements.Correct Answer: Ease of surgical excision
1414. (1767) Q4-2165:
Recklinghausenâs disease is associated with all of the following except:
1) Diffuse neurofibromatosis
3) High potential for malignant degeneration
2) Schwannoma
5) None of the above
4) Plexiform growth
Diffuse neurofibromatosis is a separate clinical presentation and is not associated with Recklinghausenâs disease.Correct Answer: Diffuse neurofibromatosis
1415. (1768) Q4-2166:
Schwannomas are differentiated from neurofibromas by all of the following except:
1) Ease of excision
3) Absence of fascicles
2) Eccentric location on the nerve
5) None of the above
4) Presence of schwann cells
Schwann cells contribute to schwannoma and neurofibroma.Correct Answer: Presence of schwann cells
1416. (1769) Q4-2167:
All of the following are true for infantile digital fibroma except:
1) 80% of infantile digital fibroma cases appear by 1 year of age
3) Intracytoplasmic inclusion bodies are present
2) Infantile digital fibroma is exclusive to fingers and toes
5) Recurrent disease never implies malignant transformation
4) Wide local excision is curative
Eighty percent of infantile digital fibromata appear before a child's first birthday. They are exclusive to the fingers and toes and are usually painless. Infantile digital fibromata are often small and the same color as the skin. On histological examination, intracytoplasmic inclusion bodies are present. Although benign, the fibromata are locally aggressive. They do not metastaaize, but recurrences after wide local excision are common. Surgery is indicated when deformity or contracture is imminent.Correct Answer: Wide local excision is curative
1417. (1770) Q4-2168:
Felon complications include all of the following except:
1) Phalangeal osteomyelitis
3) Collar button abscess
2) Suppurative flexor tenosynovitis
5) Nailbed deformity
4) Distal interphalangeal joint septic arthritis
Felons that are chronic or neglected may penetrate adjacent structures such as the distal phalanx, nailbed, or distal interphalangeal joint. They can also contribute to the formation of a pyogenic flexor tenosynovitis. Collar button abscesses are localized to web space. They typically arise from direct inoculation, not from distant felons.Correct Answer: Collar button abscess
1418. (1771) Q4-2169:
Which of the following is not a classic Kanavel sign of flexor tenosynovitis:
1) Pain on passive extension
3) Tenderness of flexor sheath
2) Flexion attitude of the finger
5) Fusiform swelling of the entire finger
4) Anesthesia of the fingertip
The cardinal signs of flexor tenosynovitis described by Kanavel include pain on passive extension, flexion attitude of the finger, tenderness of flexor sheath, and swollen finger.Correct Answer: Anesthesia of the fingertip
1419. (1772) Q4-2170:
Septic flexor tenosynovitis may involve all of the following areas except the:
1) Radial bursa
3) Paronaâs space
2) Thenar space
5) Ulnar bursa
4) Snuffbox
The radial and ulnar bursae are extensions of the tendon sheaths of the flexor pollicis longus and the flexor digitorum profundus of the small fingers. They can easily be involved in a case of pyogenic flexor tenosynovitis. Although not direct extensions of the flexor sheaths, the thenar space and Parona's space are adjacent to the flexor sheaths and can be involved in suppurative conditions. The snuffbox, however, does not have any contributions from the flexor system and is not usually involved in cases of pyogenic flexor tenosynovitis.Correct Answer: Snuffbox
Regarding the management of web space abscess, which of the following statements is not true:
1) Transverse incisions should be used.
3) Drains are often used following evacuation.
2) Wounds may be left open.
5) Thorough debridement of the dead muscle is required.
4) Early motion must be encouraged.
Transverse incisions can lead to contractures that limit finger abduction. Leaving wounds open allow for continued drainage. If preferred, closed suction drains can be used after closure of the wound. All devitalized tissue must be debrided and all signs of infection removed and irrigated copiously. Early motion is encouraged to prevent stiffness.Correct Answer: Transverse incisions should be used.
1421. (1774) Q4-2172:
Meleneyâs infection is a:
1) Spreading ulcer rimmed with gangrenous skin
3) Dry gangrene with superimposed infection
2) Patchy gangrenous involvement of the hand
5) Creates a sinus fistula to the midcarpal space
4) Multiple infective ulcer of the forearm
Found in necrotizing fasciitis, Meleneyâs infection is a spreading ulcer rimmed with gangrenous skin. The affected area must be debrided immediately. Cultures are taken at the time of surgery to tailor antibiotic coverage. Amputation is not unusual to control the spread of the gangrenous infection.Correct Answer: Spreading ulcer rimmed with gangrenous skin
1422. (1775) Q4-2173:
Meleneyâs infection is caused by:
1) Streptococcus viridans
3) Microaerophyllic non-hemolytic streptococci
2) Aerobic hemolytic staphylococci
5) Staphylococcus aureus
4) B & C
Aerobic hemolytic staphylococci and microaerophyllic non-hemolytic streptococci synergistically act to produce Meleneyâs infection. Meleney's infection is a gangrenous infection that often results after a small injury. The infection is characterized by significant, rapid swelling with gangrenous changes.Correct Answer: B & C
1423. (1776) Q4-2174:
The most common pathogen for osteomyelitis of phalanges is:
1) Staphylococcus aureus
3) Haemophilus influenzae
2) Streptococci
5) Pasturella multocida
4) Mix of gram-negative and gram-positive organisms
Staphylococcus aureus is the most common pathogen that causes osteomyelitis in the hand. Most cases of osteomyelitis in the hand are due to direct extension. Other pathogens can be found if there is a contaminated injury that penetrates directly into the bone. H. infuenza , mixed pathogens, and Pasturella multocida are less likely causes of osteomyelitis and are often caused by direct inoculation injuries or bites.Correct Answer: Staphylococcus aureus
The most common pathogen causing septic arthritis in the hand is:
1) Staphylococcus aureus
3) Haemophilus influenzae
2) Streptococci
5) Atypical mycobacterium
4) Mix of gram-negative and gram-positive organisms
Staphylococcus aureus is the most common pathogen that causes septic arthritis in the hand. The second most common pathogen is streptococcus species infections, which are often the result of trauma. Treatment includes incision and drainage with copius irrigation.Correct Answer: Staphylococcus aureus
1425. (1778) Q4-2176:
"Collar button" abscess refers to:
1) Web space infection
3) Extension of infection from mid-palmar space to Paronaâs space in the forearm
2) Finger pulp infection
5) Septic joint with dorsal and palmar extension
4) Eponychial infection
Collar button abscess is an infection of web space and is usually a result of penetrating trauma. Treatment of such abscesses requires incision and drainage through dorsal and palmar incisions. Care must be taken to avoid the neurovascular bundles. Finger pulp infections are known as felon. Infections involving Parona's space is typically involved in a horseshoe abscess.
Eponychial infections are limited to the nail fold. Collar button abscesses do not include joint involvement.Correct Answer: Web space infection
1426. (1779) Q4-2177:
A 35-year-old woman is bitten on her left index finger by a snake in her backyard. Management of snake bites includes all of the following except:
1) Keeping the patient emotionally and physically still
3) Identifying the snake
2) Applying a tourniquet
5) Call immediately for help
4) Injecting antivenin locally based on recommended guidelines
There are different snake bite protocols depending on the species of snake. However, common steps in all snake bite protocols include keeping the patient emotionally and physically still, calling for help immediately, applying a moderately tight tourniquet proximally to prevent further spread of venom, and capture or identification of the snake. Local injection of the antivenin in the fingers or toes is contraindicated.Correct Answer: Injecting antivenin locally based on recommended guidelines
1427. (1780) Q4-2178:
Cardinal signs of evenomation include all of the following except:
1) Fang marks
3) Pain
2) Cyanosis
5) Ascending lymphangitis
4) Swelling
The cardinal signs of evenomation appear between 10 minutes and 4 hours after a person is bitten. The signs include fang marks, pain, swelling, and local necrosis. Cyanosis is not considered a cardinal sign of evenomation.Correct Answer: Cyanosis
All of the following nerves are involved in infection with Mycobacterium leprae except the:
1) Ulnar nerve at the elbow
3) Supraorbital nerve
2) Median nerve in the carpal tunnel
5) Spinal accessory nerve
4) Vagus nerve
Mycobacterium leprae causes skin, nerve, and tendon sheath infections. M leprae commonly affects the hands because it has a predilection for cool parts of the body. M leprae causes neuropathy, which frequently involves the ulnar nerve at the elbow and the median nerve at the wrist. The resulting limb deformities require various surgical procedures. Cranial nerves and autonomic nerves are not affected.Correct Answer: Vagus nerve
1429. (1782) Q4-2180:
A 24-year-old white man presents to the emergency department. He was bitten on his fist while fighting with another man. You notice teeth marks on the dorsum of the metacarpophalangeal (MCP) joint of the right middle finger. The bite does not appear to be deep because the joint is not exposed, and you can see the extensor tendon, which seems intact. The patient has active extension at the MCP joint. The wound is red and swollen, but there is no tenderness or redness on the volar aspect of the MCP joint. The patient has some limitation in range of motion. He is afebrile. Radiographs reveal air in the joint but no joint dislocation or fracture, and there is no neurovascular deficit. All of the following are appropriate steps in the management of this patient except:
1) Injection of tetanus toxoid
3) Exploration for air in the joint
2) Closure of the wound
5) Splinting
4) Admitting the patient for observation and intravenous antibiotics
Human bite wounds on the hand are typically found over the MP joint. The mechanism of injury is a clenched-fist blow to the mouth. Oral flora enters the wound, which often communicates with the joint. Eikenella corrodens is frequently cultured from human bite wounds, but the most common pathogen is staphylococcus aureus. Appropriate treatment includes the administration of tetanus toxoid, exploration if there is air in the joint or frank infection, observation, intravenous antibiotics, arm elevation, and splinting. All bites over joints should be assumed to penetrate and require formal incision and drainage.Correct Answer: Closure of the wound
1430. (1783) Q4-2181:
A 24-year-old man presents to the emergency department. He was bitten on his fist while fighting with another man. You notice teeth marks on the dorsum of the metacarpophalangeal (MCP) joint of the right middle finger. The bite does not appear to be deep because the joint is not exposed, and you can see the extensor tendon, which seems intact. The patient has active extension at the MCP joint. The wound is red and swollen, but there is no tenderness or redness on the volar aspect of the MCP joint. The patient has some limitation in range of motion. He is afebrile. Radiographs reveal soft tissue involvement but no joint dislocation or fracture, and there is no neurovascular deficit. An important step in assessment of human bites is:
1) Evaluation for tendon injury in clenched-fist position
3) Ultrasound to rule out septic arthritis
2) Bone scan to rule out osteomyelitis
5) None of the above
4) Monitoring finger girth to document progress
Evaluation for tendon injury in a clenched-fist position is essential because tendons slide proximally in the open-hand position. Involvement of tendon or joint usually necessitates surgical debridement.Correct Answer: Evaluation for tendon injury in clenched-fist position
A 24-year-old white man presents to the emergency department. He was bitten on his fist while fighting with another man. You notice teeth marks on the dorsum of the metacarpophalangeal (MCP) joint of the right middle finger. The bite does not appear to be deep because the joint is not exposed, and you can see the extensor tendon, which seems intact. The patient has active extension at the MCP joint. The wound is red and swollen, but there is no tenderness or redness on the volar aspect of the MCP joint. The patient has some limitation in range of motion. He is afebrile. Radiographs reveal soft tissue involvement but no joint dislocation or fracture, and there is no neurovascular deficit. After cultures are taken, the next important step in treatment is:
1) Debridement in the emergency department and suture
3) Admit and administer IV antibiotics
2) Single dose intravenous (IV) antibiotics and discharge on oral antibiotics with follow-up instructions
5) None of the above
4) Patient work up for human immunodeficiency virus
A patient with a human bite must be admitted for IV antibiotics and observation. If left untreated, human bites are commonly infected by a mixed flora of organisms. Therefore, they must be treated diligently.Correct Answer: Admit and administer IV antibiotics
1432. (1785) Q4-2183:
A 24-year-old white man presents to the emergency department. He was bitten on his fist while fighting with another man. You notice teeth marks on the dorsum of the metacarpophalangeal (MCP) joint of the right middle finger. The bite does not appear to be deep because the joint is not exposed, and you can see the extensor tendon, which seems intact. The patient has active extension at the MCP joint. The wound is red and swollen, but there is no tenderness or redness on the volar aspect of the MCP joint. The patient has some limitation in range of motion. He is afebrile. Radiographs reveal soft tissue involvement but no joint dislocation or fracture, and there is no neurovascular deficit. The most appropriate antibiotic treatment includes:
1) Imipenam and ciprofloxacin
3) Amoxicillin and ciprofloxacin
2) Cefotaxime and ciprofloxacin
5) Bactrim and rifampin
4) Amoxicillin and flocloxacillin
Imipenam and ciprofloxacin provide treatment for gram-negative and gram-positive organisms.Correct Answer: Imipenam and ciprofloxacin
1433. (1786) Q4-2184:
Which of the following organisms is most likely found in a cat bite:
1) Eikenella corrodens
3) Micrococcus
2) Pasteurella multocida
5) None of the above
4) Borrelia recurrentis
Pasteurella multocida is the most common organism found in animal bites.Correct Answer: Pasteurella multocida
1434. (1787) Q4-2185:
Which of the following is the atypical mycobacterium that infects a penetrating wound sustained in an aquatic environment:
1) Mycobacterium avium
3) Mycobacterium aquaticum
2) Mycobacterium marinum
5) Mycobacterium chelorei
4) Mycobacterium tuberculosis hominis
Tuberculosis is the most common chronic infection found in the hand. Mycobacterium marinum is the atypical mycobacterium that can infect a wound sustained in a marine environment, freshwater lake, or tropical fish tanks. It is also called swimming pool granuloma or fish tank granuloma.Correct Answer: Mycobacterium marinum
Which of the following is not true for infections caused by Mycobacterium marinum:
1) Noncaseating granuloma is present.
3) Lowenstein-Jensen media can be used for cultures.
2) Minocycline is the preferred treatment.
5) It is also referred to as fish tank granuloma.
4) Painful swelling of digit, palm, or wrist is present with redness, warmth, and tenderness.
Tuberculous infections are chronic infections and do not produce acute signs of inflammation. Therefore, pain and tenderness are present in these infections but warmth and redness are absent. Abscesses produced in tuberculous infections are termed "cold abcesses."Correct Answer: Painful swelling of digit, palm, or wrist is present with redness, warmth, and tenderness.
1436. (1967) Q4-2379:
Which of the following fascial structures does not contribute to the formation of the spiral cord:
1) Pretendinous band
3) Graysonâs ligament
2) Lateral digital sheet
5) Spiral band
4) Clelandâs ligament
The pretendinous band, spiral band, lateral digital sheet, and Grayson's ligament are all parts of the normal fascia that contribute to the formation of the spiral cord. Cleland's ligament is not involved at all in the disease process.Correct Answer: Clelandâs ligament
1437. (1968) Q4-2380:
Which of the following structures contribute to the formation of the spiral cord:
1) Pretendinous band, Graysonâs ligament, and Clelandâs ligament
3) Pretendinous band, Landsmeer ligament, and Graysonâs ligament
2) Pretendinous band, lateral digital sheet, and Graysonâs ligam
5) Cleland's ligament and Grayson's ligament
4) Pretendinous band, Landsmeer ligament, and natatory ligament
The spiral cord is formed from the pretendinous band, spiral band, Grayson's ligament, and lateral digital sheet. Cleland's ligament is not affected in Dupuytren's contracture. Remember that bands and ligaments give rise to cords, which are the diseased state.Correct Answer: Pretendinous band, lateral digital sheet, and Graysonâs ligam
1438. (1969) Q4-2381:
Surgical release in Dupuytrenâs disease is indicated in which of the following:
1) A 15 metacarpophalangeal (MP) joint contracture and a 15 proximal interphalangeal (PIP) joint contracture
3) Palpable spiral cord involving ring and small fingers
2) A 15 MP joint contracture and a 0 PIP joint contracture
5) Palpable spiral cord involving ring and small fingers in both of the hands
4) Palpable spiral cord involving ring and small fingers with multiple skin pits
The indication for surgery in Dupuytrenâs disease is MP joint contracture larger than 30 and any degree of PIP joint deformity. The other answers are incorrect because they are subjective descriptions and do not necessarily indicate joint involvement.Correct Answer: A 15 metacarpophalangeal (MP) joint contracture and a 15 proximal interphalangeal (PIP) joint contracture
Skin pits in Dupuytrenâs disease are caused by:
1) Vertical septae of Legueu and Juvara
3) Longitudinal fibers of palmar aponeurosis inserting into the skin
2) Vertical fibers of palmar aponeurosis anchoring to the skin
5) Contractures of the natatory ligaments
4) Longitudinal pretendinous bands
The longitudinal fibers forming layer 1 of the palmar aponeurosis insert into the dermis and, when contracted, give rise to skin pits. The pretendinous bands give rise to the central cord. Contractures of the natatory ligament give rise to the natatory cord. Vertical fibers and septae do not give rise to pits.Correct Answer: Longitudinal fibers of palmar aponeurosis inserting into the skin
1440. (1971) Q4-2383:
An otherwise healthy man has Dupuytrenâs disease, which involves his small finger with 40 proximal interphalangeal joint involvement. The preferred surgery in this patient is:
1) Partial fasciectomy
3) Arthroplasty
2) Arthrodesis
5) Fasciotomy with skin grafting
4) Osteotomy
A partial fasciectomy is the preferred surgery in this situation. It is recommended that involved fascial cords be resected rather than performing prophylactic fasciectomies. Care must be taken to preserve the neurovascular bundles. Arthrodesis, arthroplasty, and osteotomies are salvage procedures meant for recurrent disease. Fasciotomy has been recommended for elderly patients who cannot tolerate a long operation.Correct Answer: Partial fasciectomy
1441. (1972) Q4-2384:
Marjolinâs ulcers are risk factors for which of the following tumors:
1) Squamous cell carcinoma
3) Melanoma
2) Basal cell carcinoma
5) Osteosarcoma
4) Synovial sarcoma
Marjolinâs ulcers are malignant degenerations in chronic skin ulcers, sinuses, and burn scars and are risk factors for squamous cell carcinoma. Any patient with chronic, nonhealing ulcers should undergo biopsy for malignant degeneration. Basal cell carcinoma, malignant melanoma, synovial sarcoma, and osteosarcoma are not commonly associated with Marjolin's ulcer.Correct Answer: Squamous cell carcinoma
1442. (1973) Q4-2385:
Which of the following tumors rarely metastasizes:
1) Malignant melanoma
3) Clear cell carcinoma
2) Synovial sarcoma
5) Squamous cell carcinoma
4) Dermatofibrosarcoma protuberans
Dermatofibrosarcoma protuberans presents as a variably colored, slow-growing nodule involving the subcutaneous tissue. This tumor has a low incidence of metastasis; however, the recurrence rate is high even with wide local excision.Correct Answer: Dermatofibrosarcoma protuberans
1) Sweat gland tumors
3) Melanoma
2) Squamous cell carcinoma
5) Merkel's cell carcinoma
4) Basal cell carcinoma
Basal cell carcinomas rarely metastasize. Malignant sweat gland tumors, malignant melanoma, and Merkel's cell carcinoma are aggressive. Treatment usually includes regional lymphadenectomy. Squamous cell carcinoma is one of the most common hand malignancies and has the capacity to metastasize via the lymphatics.Correct Answer: Basal cell carcinoma
1444. (1975) Q4-2387:
Which of the following is not a characteristic of an acrolentiginous melanoma:
1) Involves palm and nail bed
3) Presents in older patients
2) Delays in diagnosis are common
5) Develops rapidly
4) Does not commonly metastasize
Acrolentiginous melanomas are usually flat, pigmented lesions. These melanomas often occur in older patients, affecting the palm and nail bed. At the time of presentation, acrolentiginous melanomas are frequently metastasized.Correct Answer: Does not commonly metastasize
1445. (1976) Q4-2388:
Nevi at greatest risk for malignant degeneration are:
1) Giant congenital hair variety
3) Junctional nevi
2) Blue nevi
5) Nevus sebaceous
4) Compound nevi
Giant congenital hairy nevi, dysplastic nevi, senile lentigo, and congenital melanocytic nevi have significant risk for malignant degeneration.
Nevus sebaceous is present at birth in the head and neck region and has a 10% incidence of malignant transformation. Blue nevi, junctional nevi, and compound nevi have lower risks for transformation.Correct Answer: Giant congenital hair variety
1446. (1977) Q4-2389:
All of the following are risk factors for malignant melanoma except:
1) Sunlight
3) Copper-based tanning lotions
2) Genetic predisposition
5) Immunosuppressive state
4) Atypical nevi
Sun exposure is the most important risk factor for melanoma. Family history, atypical nevi, and an immunosuppressive state increases an individual's risk for developing melanoma. It is unknown if tanning lotions cause malignant melanomas.Correct Answer: Copper-based tanning lotions
1) Soft tissue sarcomas frequently metastasize to bone.
3) Soft tissue sarcomas present as a painful mass in the hand.
2) Patients with soft tissue sarcomas of the hand have worse prognosis than patients with similar tumors in other extremities.
5) Soft tissue sarcomas can be "shelled out".
4) Overall prognosis is better with radical resection than wide excision.
Soft tissue sarcomas in the hand present as painless lesions, do not metastasize to bone, and although local control is better with radical resection, there is no improvement in overall survival when compared to wide excision. Soft tissue sarcomas should not be "shelled out."Correct Answer: Patients with soft tissue sarcomas of the hand have worse prognosis than patients with similar tumors in other extremities.
1448. (1979) Q4-2391:
Which of the following tumor metastasizes to lungs:
1) Squamous cell carcinoma
3) Soft tissue sarcoma
2) Basal cell carcinoma
5) Actinic keratosis
4) Schwannoma
Squamous cell carcinoma and basal cell carcinoma do not commonly metastasize to the lungs. Actinic keratoses are premalignant lesions that progress into squamous cell carcinomas. Schwannomas are common benign nerve tumors.Correct Answer: Soft tissue sarcoma
1449. (1980) Q4-2393:
Which of the following is not a characteristic of synovial sarcomas:
1) Synovial sarcomas are poorly differentiated masses located close to joints, tendon, or bursa.
3) Lymphatic spread is common.
2) Spindle and epithelial-type cells with monophasic or biphasic cells are present on histology.
5) High grade malignant soft tissue sarcoma
4) Wide or radical excision with radiation or chemotherapy prevents metastases.
Synovial sarcomas are high grade malignant soft tissue sarcomas, in which metastases can occur years after surgery. Long term followup is necessary. They arise close to joints, tendons or bursa and lymphatic spred is common. Histology reveals spindle and epithelial type cells with menophasic or biphasic pattern. Treatment includes wide resection and radiation, chemotherapy is not usually used.Correct Answer: Wide or radical excision with radiation or chemotherapy prevents metastases.
1450. (1981) Q4-2394:
Characteristic histological features of malignant schwannoma are best described as:
1) Fusiform cells with neoplastic schwann cells and nerve fascicles
3) Round or fusiform cells with clear cytoplasm and nerve fascicles
2) Spindle and epithelial-type cell mix with monphasic or biphasic cells
5) None of the above
4) Pleomorphic spindle histiocytes and giant cells in a storiform pattern
The histological features of malignant schwannoma have characteristic fusiform cells with neoplastic schwann cells and nerve fascicles.Correct Answer: Fusiform cells with neoplastic schwann cells and nerve fascicles
1) Megalodactyly
3) Gigantism
2) Overgrowth
5) Macrodactylia fibrolipomatosis
4) Symbrachydactyly
Symbrachydactyly is a term that encompasses all variations of shortened digits. Symbrachydactyly may be associated with syndactylies, but it is not a term used in reference to macrodactyly.Correct Answer: Symbrachydactyly
1452. (2164) Q4-2590:
Most cases of macrodactyly are:
1) Bilateral and affect men more often than women
3) Unilateral and affect men more often than women
2) Bilateral and affect women more often than men
5) Bilateral with equal frequency in both men and women
4) Unilateral and affect women more often than men
The majority of patients (90%) present with unilateral macrodactyly, and men are more often affected than women. Macrodactyly is most frequently found in the index finger, followed by the long finger, thumb, ring, and little fingers. Typically, two digits are affected â most commonly the thumb and index or the index and long.Correct Answer: Unilateral and affect men more often than women
1453. (2165) Q4-2591:
Which digit is most commonly affected by macrodactyly:
1) Thumb
3) MIddle
2) Index
5) Small
-
Ring
The index finger is most frequently affected, although multiple digital enlargement is actually more commonly seen.Correct Answer: Index
1454. (2166) Q4-2592:
Syndactyly is present in what percentage of patients with macrodactyly:
1) 10%
3) 30%
2) 20%
-
50%
4) 40%
The majority of patients (90%) present with unilateral macrodactyly, and men are more often affected than women. Macrodactyly is most frequently found in the index finger, followed by the long finger, thumb, ring, and little fingers. Typically, two digits are affected â most commonly the thumb and index or the index and long. Syndactyly may be present in 10% of patients with macrodactyly.Correct Answer: 10%
-
Birth advancing macrodactyly (BAM)
3) Progressive macrodactyly
2) Static macrodactyly
5) Complex macrodactyly
4) Simple macrodactyly
Barsky described macrodactyly as either static or progressive. Static macrodactyly is present at birth, and the affected digit grows larger as the child develops. In the progressive type of macrodactyly, growth begins soon after birth. This form of the disorder is more common than static macrodactyly.Correct Answer: Static macrodactyly
1456. (2168) Q4-2594:
The most accepted theory for the cause of macrodactyly is:
1) Idiopathic
3) Vascular
2) Neural
5) Congenital
4) Humoral
Some surgeons believe that macrodactyly is a variant of neurofibromatosis. Although macrodactyly is not an inherited anomaly, there are syndromes that may be associated with enlarged digits such as Proteus syndrome. Although numerous causes have been suggested, the most accepted theory was described by Inglis in 1950. He theorized that the abnormal nerves exert influence on the local tissues to stimulate growth.Correct Answer: Neural
1457. (2169) Q4-2595:
Syndromes that may be associated with macrodactyly include:
1) Proteus syndrome
3) Madelungâs deformity
2) Freeman-Sheldon syndrome
5) Poland syndrome
4) Holt-Oram syndrome
Some surgeons believe that macrodactyly is a variant of neurofibromatosis. Although macrodactyly is not an inherited anomaly, there are syndromes that may be associated with enlarged digits such as Proteus syndrome. Theoretical causes for macrodactyly include a neural cause, a vascular cause, as well as a humoral mechanism. The most accepted theory is that abnormal nerves exert some influence on the local tissues to stimulate growth.Correct Answer: Proteus syndrome
1458. (2170) Q4-2596:
Macrodactyly affects:
1) Only bones
3) Bones, fat, and nerves
2) Bones and fat
5) Bones, fat, nerves, blood vessels, and tendons
4) Bones, fat, nerves, and blood vessels
Although this is controversial, the majority of surgeons believe that macrodactyly affects bones, fat, and nerves.Correct Answer: Bones, fat, and nerves
enlarged ring and small fingers. There is full range of motion without instability. After examination of the patient, you recommend:
1) Scheduling the patient for immediate surgery
3) Performing surgery within 1 week of diagnosis
2) Telling the parents to return when the child develops functional abnormalities
5) Performing additional testing
4) Scheduling surgery to coincide with the patient beginning school
The child is not ready for surgery. Although surgery may coincide with the patient beginning school, this does not always occur. At this time, additional examination and testing are recommended.Correct Answer: Performing additional testing
1460. (2172) Q4-2598:
A 2-year-old child is brought to your office for evaluation of a "big hand." Upon examination, you notice that the child has mildly enlarged ring and small fingers. There is full range of motion without instability. The childâs parents inform you that they would like you to amputate the affected digits as soon as possible. You should:
1) Begin radiation therapy to arrest the growth of the affected digits
3) Send the patient to another hand surgeon
2) Proceed with amputation because you have the parentsâ consent
5) Schedule the patient for a debulking procedure
4) Explain the typical course of macrodactyly and order additional testing
Although amputation may be necessary in some patients with macrodactyly, it is too early in the course of this case to begin entertaining such a drastic measure. A debulking procedure is not recommended for a 2-year-old child. Radiation therapy is not an option in uncomplicated cases of macrodactyly. The surgeon must educate the parents about the disease process and order additional testing.Correct Answer: Explain the typical course of macrodactyly and order additional testing
1461. (2173) Q4-2599:
You discover that a patient who you have been treating for macrodactyly has been followed by the Proteus Syndrome Foundation. Exhaustive work-up has been completed and radiographs of the hand reveal:
1) Multiple enchondromas in the affected fingers
3) Enlarged bones in length and width
2) Enlarged bones in length only
5) Normal appearing bones
4) Enlarged bones in width only
In patients with macrodactyly, surgeons do not typically find enchondromas, especially not multiple enchondromas in the affected fingers. Enlargement of the bones is found in all dimensions â not only in the length and width. If the bones appear normal on radiograph, then they are not affected by macrodactyly.Correct Answer: Enlarged bones in length and width
1462. (2174) Q4-2600:
A 2-year-old child is brought to your office for evaluation of a "big hand." Upon examination, you notice that the child has mildly enlarged ring and small fingers. There is full range of motion without instability. After examination of the patient, you discuss the diagnosis of macrodactyly with the parents. The parents feel assured after your discussion of the disease process and your review of the radiographs. You should next see the patient:
1) Never
3) When functional abnormalities develop
2) In 1 year
5) When the patient is old enough to consent for surgery
4) When the enlargement of the digits has ceased
Patients with macrodactyly should be followed up yearly. Although the parents may be difficult, this is not a reason to stop seeing a patient. The other answers choices are incorrect because treatment would be too late.Correct Answer: In 1 year
1) Nerve injury
3) Decreased sensation
2) Bony malunion
5) Infection
4) Joint laxity
Complications of macrodactyly surgery include poor healing of flaps secondary to devascularization or undue tension, nerve injury or decreased sensation, infection, stiffness, bony nonunion or malunion, and failure of the epiphysiodesis.Correct Answer: Joint laxity
1464. (2176) Q4-2602:
Epiphysiodesis for macrodactyly should be performed at the following location:
1) Proximal phalanx only
3) Proximal phalanx, middle phalanx, and distal phalanx
2) Proximal phalanx and middle phalanx
5) Proximal phalanx and distal phalanx
4) Middle phalanx only
Treatment by epiphysiodesis for macrodactyly is ineffective if only single phalanges are treated. Therefore, treatment of the proximal phalanx, distal phalanx, or the middle phalanx alone is incorrect. The author prefers to perform epiphysiodesis only on the proximal and distal phalanges. The middle phalanx is not treated to preserve motion at the proximal interphalangeal joint.Correct Answer: Proximal phalanx and distal phalanx
1465. (2187) Q4-2614:
When ruptured, which portion of the scapholunate ligament leads to scaphoid-lunate diastasis:
1) Distal
3) Intermediate
2) Proximal
5) Volar
4) Dorsal
The dorsal section of the scapholunate ligament is the strongest portion, requiring 300 N of load for failure. The volar (150 N) and intermediate portions (25 N to 50 N) contribute less to overall stability.Correct Answer: Dorsal
1466. (2188) Q4-2615:
Which of the following radiographic views is not routinely used to diagnose scapholunate injury:
1) Semisupination oblique view
3) Lateral view
2) Clenched fist view
5) Oblique view
4) Anteroposterior (AP) view
The semisupination oblique view is used to visualize the pisiform and pisotriquetral joint. The PA oblique and lateral views are the primary films used to diagnose scapholunate instability. The clenched fist view is used as a provocative view to bring out dynamic instability.Correct Answer: Semisupination oblique view
1) Open repair with bone sutures
3) Closed reduction and long arm cast
2) Proximal row carpectomy
5) Open repair with suture anchors
4) Arthroscopically assisted reduction and pinning
Proximal row carpectomy is a salvage procedure for chronic instability with focal radioscaphoid arthritis. Open repair with sutures through bone tunnels, open repair with suture anchors, and arthroscopically assisted reduction and pinning have been used successfully in acute cases.Correct Answer: Proximal row carpectomy
1468. (2190) Q4-2617:
Which of the following is considered indicative of a scaphoid-lunate ligament tear on posteroanterior radiograph:
1) Terry Thomas sign
3) Spilled tea cup sign
2) Volar intercalated segmental instability (VISI) pattern
5) Dorsal intercalated segment instability (DISI) pattern
4) Watson-Jones scaphoid shift
The VISI, DISI, and spilled tea cup signs are seen on lateral radiographs, whereas the Watson-Jones scaphoid shift test is a clinical sign. The classic pattern after scaphoid-lunate ligament injury is a DISI pattern as the lunate extends and the scaphoid flexes. The spilled tea cup sign is present in perilunate dislocations.Correct Answer: Terry Thomas sign
1469. (2191) Q4-2618:
The Terry Thomas sign, which is considered indicative of scaphoid-lunate ligament rupture, is best described as:
1) Scapholunate diastases larger than 3 mm
3) Reduction and exaggeration of scapholunate diastases on radial and ulnar deviation motion studies
2) Scapholunate angle more than 30º to 60º on lateral radiographs
5) Scaphoid flexion with lunate extension
4) Exaggeration and reduction of scaphoid-lunate diastases on radial and ulnar deviation radiographs, respectively
The Terry Thomas sign refers to scapholunate diastases that may be apparent on posteroanterior radiographs of the wrist and is indicative of rupture if the diastases are larger than 3 mm. It is named after the famous comedian who had a gap between his front teeth.Correct Answer: Scapholunate diastases larger than 3 mm
1470. (2192) Q4-2619:
The most important requirement for a diagnostic magnetic resonance image (MRI) study in cases of scaphoid-lunate ligament injury is:
1) 2 mm thin slices
3) Gallium-enhanced scan
2) Tangential cuts
5) MRI in neutral, radial, and ulnar deviation
4) Dedicated wrist coil
MRI is not considered the technique of choice for the evaluation of the scaphoid-lunate ligament. Standard MRI coils are not adequate for the evaluation of the ligaments of the wrist. To maximize the yield from a wrist MRI, high-field strength and high-resolution images must be obtained using dedicated wrist coils. Only with such dedicated coils can detailed information be derived regarding the continuity of the scapoid-lunate ligament. Physical examination and wrist arthroscopy remain the gold standards for the evaluation of a torn scaphoid-lunate ligament.Correct Answer: Dedicated wrist coil
1) Herbert screw (reduction association of the scapholunate)
3) Scaphoid-lunate ligament reconstruction using bone-ligament-bone autograft
2) Scaphotrapeziotrapezoid (STT) fusion
5) Repair with capsulodesis
4) Allograft ligament
In cases of subacute scaphoid-lunate ligament injury without arthrosis, it is acceptable to attempt reconstruction with bone anchors, allograft ligament repair, capsulodesis, bone-ligament-bone autograft, and the RASL procedure with a Herbert screw. In the presence of localized arthritis, one might consider one of the limited wrist fusions such as scaphotrapeziotrapezoid fusion.Correct Answer: Scaphotrapeziotrapezoid (STT) fusion
1472. (2317) Q4-2770:
Mallet finger injuries refer to:
1) Fractures of the bony tuft
3) Lack of conjoined tendon continuity at the distal interphalangeal (DIP) joint
2) Flexor tendon injuries
5) Intrinsic tightness
4) Fractures of the middle phalanx
Mallet finger injuries may be associated with fractures of the bony tuft, fractures of the middle phalanx, flexor tendon injuries, and intrinsic tightness. However, mallet injuries refer to lack of continuity at the DIP joint.Correct Answer: Lack of conjoined tendon continuity at the distal interphalangeal (DIP) joint
1473. (2318) Q4-2772:
In mallet finger injuries, the distal phalanx posture is:
1) Hyperextended
3) Neutral
2) Flexed
5) Ulnarly deviated
4) Radially deviated
The characteristic deformity is âdroopingâ at the distal interphalangeal (DIP) joint. The DIP is flexed. It is not hyperextended, neutral, or deviated.Correct Answer: Flexed
1474. (2319) Q4-2773:
Mallet finger injuries are typically:
1) Secondary to hyperextension injuries
3) Secondary to torsion injuries
2) Secondary to forced flexion injuries
5) Asymptomatic
4) Secondary to fingertip amputations
Mallet finger usually results from a blow to the tip of the extended finger. This forces distal phalanx flexion and disruption of the extensor mechanism at the distal interphalangeal joint. Open injuries to the extensor mechanism can also cause mallet finger.Correct Answer: Secondary to forced flexion injuries
1) Cast immobilization of the affected digit in extension
3) Early active motion of the affected joint
2) Dorsal block splint of the affected digit
5) Splinting of the affected DIP joint in extension
4) Splinting of the affected distal interphalangeal joint (DIP) joint in flexion
Cast immobilization is excessive and will cause undue stiffness in the affected finger. Dorsal blocking splints, splinting in flexion, and early active motion are contraindicated in these injuries. Only the affected joint should be splinted in extension.Correct Answer: Splinting of the affected DIP joint in extension
1476. (2321) Q4-2775:
Type I mallet finger injuries must be immobilized constantly for a minimum of:
1) 4 weeks
3) 6 weeks
2) 5 weeks
5) 8 weeks
4) 7 weeks
Eight weeks of immobilization is preferred. If the finger is immobilized for a shorter period of time, the clock is reset and immobilization is started again.Correct Answer: 8 weeks
1477. (2323) Q4-2777:
The most common mallet finger injuries are:
1) Type I
3) Type III
2) Type II
5) Type V
4) Type IV
Type I mallet injuries are by far the most common mallet injuries. There is no such classification as a type V injury.Correct Answer: Type I
1478. (2324) Q4-2778:
On physical examination, a mallet finger assumes a:
1) Resting flexed posture with active and passive extension
3) Resting flexed posture without active extension
2) Resting flexed posture without passive extension
5) Resting flexed posture with active extension
4) Resting flexed posture without active or passive extension
The distal phalanx assumes a resting flexed posture. The patient is not able to actively extend the fingertip, but it can be passively extended.Correct Answer: Resting flexed posture without active extension
1479. (2325) Q4-2779:
The following mallet finger injuries always require tendon repair:
1) Type I and type II
3) Type III and type IV
2) Type II and type III
5) Type I and type IV
4) Type IV and type V
Type II and III injuries have absolute requirements for tendon repair as there is a laceration or loss of tendon substance.Correct Answer: Type II and type III
1480. (2326) Q4-2780:
After placing a type I mallet finger in a splint at the initial visit, next follow-up should be:
1) The following day
3) In 2 weeks
2) In 1 week
5) At the end of the 8-week regimen
4) In 1 month
After placement of the splint, the patient should follow-up in the next week to make sure the finger is still maintained in full extension. Loosening of the splint will occur as swelling decreases.Correct Answer: In 1 week
1481. (2335) Q4-2792:
The most common bone tumor of the upper extremity is:
1) Enchondroma
3) Osteochondroma
2) Osteoblastoma
5) Chondromyxoid tumor
4) Giant cell tumor
Osteochondromas are the most common primary benign bony tumors.Correct Answer: Osteochondroma
1482. (2336) Q4-2793:
The most common benign bone tumor of the hand is:
1) Enchondroma
3) Osteochondroma
2) Osteoblastoma
5) Chondromyxoid tumor
4) Giant cell tumor
Unlike the entire upper extremity, enchondromas are the most common tumors of the hand.Correct Answer: Enchondroma
1483. (2337) Q4-2794:
Osteochondromas are benign but can have a malignant transformation in which of the following cases:
1) Diaphyseal achalasia
3) Osteochondromatosis malignant transformans
2) Ollierâs disease
5) Mafucci's syndrome
4) Osteochondroma larger than 5 cm
Diaphyseal achalasia, also known as multiple hereditary exostoses, has a risk of malignant degeneration in up to 25% patients. Ollierâs disease and Mafucciâs syndrome are associated with enchondromas. There is no lesion called an osteochondromatosis malignant transformans.Correct Answer: Diaphyseal achalasia
1484. (2338) Q4-2795:
The risk of malignant transformation in patients with multiple hereditary exostoses is:
1) 0%
3) 1% to 2%
2) Less than 1%
5) 0.5% to 25%
4) Greater than 5%
The rate of malignant transformation in patients with multiple hereditary exostoses is variable and is generally reported between 0.5% to 25%.Correct Answer: 0.5% to 25%
1485. (2339) Q4-2796:
Recurrence of osteochondroma is likely if:
1) The cartilage cap is incompletely excised
3) The bony stalk is incompletely excised
2) The overlying bursa is incompletely excised
5) Its connection with the medullary canal is not obliterated
4) The tumor is incompletely excised
The cartilaginous portion of an osteochondroma is the neoplastic part; its complete excision is essential to avoid recurrences.Correct Answer: The cartilage cap is incompletely excised
1486. (2340) Q4-2797:
Malignant transformation of osteochondroma commonly occurs to:
1) High-grade osteosarcoma
3) Low-grade chondrosarcoma
2) High-grade chondrosarcoma
5) Parosteal osteosarcoma
4) Low-grade osteosarcoma
Osteochondroma is a cartilaginous tumor and malignant transformation is to a low-grade chondrosarcoma.Correct Answer: Low-grade chondrosarcoma
1487. (2341) Q4-2798:
All of the following suggest a possibility of malignant transformation in multiple hereditary exostoses except:
1) Recent onset of pain
3) Cartilaginous cap thickness greater than 3 cm
2) Growth after skeletal maturity
5) Calcific stippling in the cap on radiograph
4) Soft tissue extension
Stippling on radiographs in the cap is due to calcification and is a common characteristic of cartilaginous tumors.Correct Answer: Calcific stippling in the cap on radiograph
1488. (2342) Q4-2799:
Enchondromas are commonly involved in which of the following sites:
1) Metacarpals
3) Radius
2) Carpus
5) Clavicle
4) Ulna
Metacarpals and phalanges are the most common areas of hand involvement, and the hand is involved in 40% to 65% of cases. Enchondromas are also the most common primary benign bone tumor of the hand (90% cases).Correct Answer: Metacarpals
1489. (2343) Q4-2800:
The most common forearm deformity in patients with hereditary multiple osteochondromatosis is:
1) Ulnar shortening
3) Radial head dislocation
2) Radial shortening
5) Translocation of carpus
4) Madelungâs deformity
Ulnar involvement and shortening frequently occurs in patients with hereditary multiples osteochondromatosis because the distal ulnar growth plate is smaller than that of the radius, consequently its length is affected more. The ulnar shortening causes radial bowing or radial head dislocation.Correct Answer: Ulnar shortening
|
Slide 1
A 12-year-old boy is brought to the clinic by his concerned parents. The boyâs forearm is bowed, and his parents are confused as to the possible diagnosis and treatment options. You notice that the right forearm of the child is bowed ulnarwards and is shorter compared to the left forearm. The pronosupination is markedly decreased on the right side but is also limited on the left side. The patient has a good grip, pinch, and grasp. He is neurologically intact as well.
The parents say that they first noticed the deformity around 6 or 7 years ago, and the mother informs you that she had noticed a hard bump on the forearm. She has recently noticed another bump on his right leg. The child does not complain of pain and is using both of his hands quite well. The parents were informed by a previous physician that the child has Madelungâs deformity and are concerned that the disease is now involving other areas of his body.
You order a radiograph of the patientâs forearm. The anteroposterior radiograph is shown (Slide). The next step is to order a:
1) Skeletal radiograph survey
3) Magnetic resonance imaging (MRI) with wrist arthrogram
2) Magnetic resonance imaging (MRI)
5) Computed tomography (CT) scan
4) Genetic evaluation
The next step is to order a skeletal survey to rule out involvement of other areas.Correct Answer: Skeletal radiograph survey
Slide 1 Slide 2 Slide 3
A 12-year-old boy is brought to the clinic by his concerned parents. The boyâs forearm is bowed, and his parents are confused as to the possible diagnosis and treatment options. You notice that the right forearm of the child is bowed ulnarwards and is shorter compared to the left forearm. The pronosupination is markedly decreased on the right side but is also limited on the left side. The patient has a good grip, pinch, and grasp. He is neurologically intact as well.
The parents say that they first noticed the deformity around 6 or 7 years ago, and the mother informs you that she had noticed a hard bump on the forearm. She has recently noticed another bump on his right leg. The child does not complain of pain and is using both of his hands quite well. The parents were informed by a previous physician that the child has Madelungâs deformity and are concerned that the disease is now involving other areas of his body.
An immediate appointment for magnetic resonance imagine (MRI) and computed tomography (CT) scan are not available, and a genetic evaluation has been carried out previously. As you await the report from the geneticist office, you decide to get a skeletal radiograph series on the patient. The radiograph of the opposite forearm (Slide 1) and right leg are shown (Slide 2).
You order a radiograph of the forearm. The anteroposterior radiograph is shown (Slide 3). Your suspected diagnosis is:
1) Diaphyseal achalasia
3) Multiple enchondromatosis
2) Madelungâs deformity
5) Infection
4) Multiple epiphyseal dysplasia
Diaphyseal achalasia, also called multiple hereditary exostoses, classically presents in a young individual with multiple sites of involvement. The more involved the disease, the more likely hand involvement becomes. Forearm involvement is also common. The radius is bowed due to the shortened ulna. The risk of radial head dislocation is higher if the radius does not bow. While infection or traumatic injury could have produced early physeal arrest as seen in the first radiograph, presence of lesions elsewhere indicates multiple hereditary exostoses and should be investigated with skeletal surveys. Multiple epiphyseal dysplasia is not a possible diagnosis as only the ulna is involved in the first radiograph and radius alone in the left forearm. No enchondromas are present.Correct Answer: Diaphyseal achalasia
Slide 1 Slide 2 Slide 3
A 12-year-old boy is brought to the clinic by his concerned parents. The boyâs forearm is bowed, and his parents are confused as to the possible diagnosis and treatment options. You notice that the right forearm of the child is bowed ulnarwards and is shorter compared to the left forearm. The pronosupination is markedly decreased on the right side but is also limited on the left side. The patient has a good grip, pinch, and grasp. He is neurologically intact as well.
The parents say that they first noticed the deformity around 6 or 7 years ago, and the mother informs you that she had noticed a hard bump on the forearm. She has recently noticed another bump on his right leg. The child does not complain of pain and is using both of his hands quite well. The parents were informed by a previous physician that the child has Madelungâs deformity and are concerned that the disease is now involving other areas of his body.
You order a radiograph of the forearm. The anteroposterior radiograph is shown (Slide 1). The childâs skeletal radiograph survey is also presented (Slide 2 and Slide 3).
The genetic pattern seen in patients with this type of presentation is:
1) Autosomal recessive
3) Sex-linked recessive
2) Autosomal dominant
5) Sporadic
4) Sex-linked dominant
Multiple hereditary exostoses is inherited in an autosomal-dominant manner with 90% penetrance.Correct Answer: Autosomal dominant
Slide 1 Slide 2 Slide 3
A 12-year-old boy is brought to the clinic by his concerned parents. The boyâs forearm is bowed, and his parents are confused as to the possible diagnosis and treatment options. You notice that the right forearm of the child is bowed ulnarwards and is shorter compared to the left forearm. The pronosupination is markedly decreased on the right side but is also limited on the left side. The patient has a good grip, pinch, and grasp. He is neurologically intact as well.
The parents say that they first noticed the deformity around 6 or 7 years ago, and the mother informs you that she had noticed a hard bump on the forearm. She has recently noticed another bump on his right leg. The child does not complain of pain and is using both of his hands quite well. The parents were informed by a previous physician that the child has Madelungâs deformity and are concerned that the disease is now involving other areas of his body.
You order a radiograph of the forearm. The anteroposterior radiograph is shown (Slide 1). The childâs skeletal radiograph survey is also presented (Slide 2 and Slide 3).
Which of the following areas is unlikely to be involved:
1) Phalanges
3) Clavicle
2) Pelvis
5) Talus
-
Femur
The clavicle is a membranous bone, and osteochondromas do not arise in membranous bones.Correct Answer: Clavicle
Slide 1 Slide 2 Slide 3
A 12-year-old boy is brought to the clinic by his concerned parents. The boyâs forearm is bowed, and his parents are confused as to the possible diagnosis and treatment options. You notice that the right forearm of the child is bowed ulnarwards and is shorter compared to the left forearm. The pronosupination is markedly decreased on the right side but is also limited on the left side. The patient has a good grip, pinch, and grasp. He is neurologically intact as well.
The parents say that they first noticed the deformity around 6 or 7 years ago, and the mother informs you that she had noticed a hard bump on the forearm. She has recently noticed another bump on his right leg. The child does not complain of pain and is using both of his hands quite well. The parents were informed by a previous physician that the child has Madelungâs deformity and are concerned that the disease is now involving other areas of his body.
You order a radiograph of the forearm. The anteroposterior radiograph is shown (Slide 1). The childâs skeletal radiograph survey is also presented (Slide 2 and Slide 3).
The chance of hand involvement in this child is:
1) 0%
3) 25%
2) 10%
-
Undetermined
4) Greater than 25%
The hand is involved in 30% to 80% of cases.Correct Answer: Greater than 25%
Slide 1 Slide 2 Slide 3
A 12-year-old boy is brought to the clinic by his concerned parents. The boyâs forearm is bowed, and his parents are confused as to the possible diagnosis and treatment options. You notice that the right forearm of the child is bowed ulnarwards and is shorter compared to the left forearm. The pronosupination is markedly decreased on the right side but is also limited on the left side. The patient has a good grip, pinch, and grasp. He is neurologically intact as well.
The parents say that they first noticed the deformity around 6 or 7 years ago, and the mother informs you that she had noticed a hard bump on the forearm. She has recently noticed another bump on his right leg. The child does not complain of pain and is using both of his hands quite well. The parents were informed by a previous physician that the child has Madelungâs deformity and are concerned that the disease is now involving other areas of his body.
You order a radiograph of the forearm. The anteroposterior radiograph is shown (Slide 1). The childâs skeletal radiograph survey is also presented (Slide 2 and Slide 3).
The most likely complication in this child is:
1) Malignant degeneration
3) Posterior interosseous neuropathy (PIN) palsy
2) Carpal translocation
5) Elbow dislocation
4) Peroneal nerve palsy
Ulnar carpal translocation occurs due to the steep radial articular angulation that occurs due to the tethering effect of a shortened ulna and is already apparent in early stages in the first radiograph. While peroneal palsy is possible due to a proximal fibula lesion, it is less common. Malignant transformation occurs, risk varies with families.Correct Answer: Carpal translocation
Slide 1 Slide 2 Slide 3
A 12-year-old boy is brought to the clinic by his concerned parents. The boyâs forearm is bowed, and his parents are confused as to the possible diagnosis and treatment options. You notice that the right forearm of the child is bowed ulnarwards and is shorter compared to the left forearm. The pronosupination is markedly decreased on the right side but is also limited on the left side. The patient has a good grip, pinch, and grasp. He is neurologically intact as well.
The parents say that they first noticed the deformity around 6 or 7 years ago, and the mother informs you that she had noticed a hard bump on the forearm. She has recently noticed another bump on his right leg. The child does not complain of pain and is using both of his hands quite well. The parents were informed by a previous physician that the child has Madelungâs deformity and are concerned that the disease is now involving other areas of his body.
You order a radiograph of the forearm. The anteroposterior radiograph is shown (Slide 1). The childâs skeletal radiograph survey is also presented (Slide 2 and Slide 3).
The difference between Madelungâs deformity and this boyâs condition is:
1) The ulna is shorter
3) There is radial head dislocation
2) The radius is shorter
5) It is not congenital
4) There is bilateral involvement
The ulna is elongated or dorsally subluxed in Madelung's deformity.Correct Answer: The ulna is shorter
Slide 1 Slide 2 Slide 3
A 12-year-old boy is brought to the clinic by his concerned parents. The boyâs forearm is bowed, and his parents are confused as to the possible diagnosis and treatment options. You notice that the right forearm of the child is bowed ulnarwards and is shorter compared to the left forearm. The pronosupination is markedly decreased on the right side but is also limited on the left side. The patient has a good grip, pinch, and grasp. He is neurologically intact as well.
The parents say that they first noticed the deformity around 6 or 7 years ago, and the mother informs you that she had noticed a hard bump on the forearm. She has recently noticed another bump on his right leg. The child does not complain of pain and is using both of his hands quite well. The parents were informed by a previous physician that the child has Madelungâs deformity and are concerned that the disease is now involving other areas of his body.
You order a radiograph of the forearm. The anteroposterior radiograph is shown (Slide 1). The childâs skeletal radiograph survey is also presented (Slide 2 and Slide 3).
All of the following are acceptable options, either alone or in combination, for management of this childâs condition, except:
1) Excision of osteochondromas
3) Hemiphyseal stapling
2) Ulnar lengthening
5) Observation
4) Radial osteotomy
Although hemiphyseal stapling is an acceptable option to correct radial articular angulation, in this boy the distal radial physis is already fused as is seen in the first radiograph.Correct Answer: Hemiphyseal stapling
Slide 1 Slide 2 Slide 3
A 12-year-old boy is brought to the clinic by his concerned parents. The boyâs forearm is bowed, and his parents are confused as to the possible diagnosis and treatment options. You notice that the right forearm of the child is bowed ulnarwards and is shorter compared to the left forearm. The pronosupination is markedly decreased on the right side but is also limited on the left side. The patient has a good grip, pinch, and grasp. He is neurologically intact as well.
The parents say that they first noticed the deformity around 6 or 7 years ago, and the mother informs you that she had noticed a hard bump on the forearm. She has recently noticed another bump on his right leg. The child does not complain of pain and is using both of his hands quite well. The parents were informed by a previous physician that the child has Madelungâs deformity and are concerned that the disease is now involving other areas of his body.
You order a radiograph of the forearm. The anteroposterior radiograph is shown (Slide 1). The childâs skeletal radiograph survey is also presented (Slide 2 and Slide 3).
Which of the following is not true regarding the possibility of malignant degeneration in this child:
1) There is a risk of up to 25%
3) Most common secondary malignancy is chondrosarcoma
2) Bone scan can differentiate
5) Malignant change occurs in adulthood
4) Risk of malignancy varies between families
Bone scan cannot differentiate between an active lesion and a malignant transformation.Correct Answer: Bone scan can differentiate
1499. (2352) Q4-2810:
Hornerâs syndrome includes all of the following except:
1) Miosis
3) Enophthalmosis
2) Anhidrosis
5) Diplopia
4) Exophthalmosis
Hornerâs syndrome is caused by disruption of sympathetic innervation and is characterized by enophthalmosis, not exophthalmosis. Other symptoms include anhidrosis, miosis, and ptosis.
Correct Answer: Exophthalmosis
1500. (2353) Q4-2811:
Axonotmesis involves injury to the:
1) Epineurium
3) Perineurium
2) Endoneurium
5) Vasonervorum
4) Axon
Axontmesis, as described in Seddonâs classification, implies injury to the axon and myelin sheath. Neurontmesis involves injury to all three layers.
Correct Answer: Axon
1501. (2354) Q4-2812:
All of the following may be seen with preganglionic lesion except:
1) Hornerâs syndrome
3) Positive histamine test
2) Hemidiaphragmatic palsy
5) Root avulsion sleeve on myelogram
4) Tinelâs sign
Tinelâs sign is seen with postganglionic lesions. Correct Answer: Tinelâs sign
1502. (2355) Q4-2813:
Weakness is not seen with root avulsion in the:
1) Rhomboids
3) Supraspinatus
2) Serratus anterior
5) Infraspinatus
4) Trapezius
The trapezius is innervated by spinal accessory nerve and thus will not be involved in a brachial plexus lesion. In the case of a preganglionic lesion, all muscles innervated by the nerve roots will be affected.
Correct Answer: Trapezius
1503. (2356) Q4-2814:
In obstetric brachial plexus injury, an indicator of plexus recovery at 3 months is the return of the:
1) Biceps muscle
3) Brachioradialis muscle
2) Triceps muscle
5) Teres major muscle
4) Latissimus muscle
Biceps recovery at 3 months is the single most important indicator of recovery in obstetric plexus palsy. Correct Answer: Biceps muscle
1) Brachial plexus neuropraxia
3) Brachial plexus neuritis
2) Erbâs palsy
5) Klumpkeâs Palsy
4) C 5, C 6 disk herniations
The involved muscles have C 5, C 6 root innervations. Positive Tinelâs sign, functioning rhomboids and serratus anterior, and the absence of Hornerâs syndrome rule out a preganglionic lesion. The EMG finding confirms the clinical finding. Subclinical involvement of any other muscle is not shown. Neuropraxia usually recovers in 6 weeks and EMG shows fibrillation, which is inconsistent with neuropraxia. Brachial plexus neuritis, Parsonage-Turner syndrome, has an acute presentation following a painful episode involving the whole arm. There is significant history of a fall in this case.
Correct Answer: Erbâs palsy
1505. (2358) Q4-2816:
A 15-year-old boy presented with inability to elevate his right shoulder and flex his elbow. He sustained a fall from an all-terrain vehicle 8 weeks ago. He landed on the right shoulder and twisted his neck. Radiographs of the skull, chest, cervical and thoracic spine, and shoulder were normal. There was no loss of consciousness, chest pain, or breathing difficulties. The patient was observed in the hospital until stable and referred for follow-up in the hand clinic at 4 weeks. An electromyelogram (EMG) was scheduled. Clinical examination revealed weakness of deltoid, supraspinatus, infraspinatus, teres minor, biceps, brachialis, brachioradialis, and extensor carpi radialis longus. The remainder of his forearm musculature was preserved and he could grasp, release, and pinch. Sensations were decreased along the distribution of the axillary nerve. There was 3-cm wasting of his arm and 2 cm of the forearm. Tinelâs sign is positive around the clavicle. Hornerâs signs are absent and his arm lies against the body. The EMG report showed fibrillation potentials in the weak muscles. The patient can now flex his elbow. When asked to demonstrate, he flexes his wrist and pronates his forearm to swing his elbow into flexion.
The level of lesion is:
1) Postganglionic C 5, C 6
3) Posterior cord injury
2) Preganglionic C 5, C 6
5) Spinal accessory paralysis
4) Middle trunk
The involved muscles have C 5, C 6 root innervations. Positive Tinelâs sign, functioning rhomboids and serratus anterior, and the absence of Hornerâs syndrome rule out a preganglionic lesion. The EMG finding confirms the clinical finding. Subclinical involvement of any other muscle is not shown. Neuropraxia usually recovers in 6 weeks and EMG shows fibrillation, which is inconsistent with neuropraxia. Brachial plexus neuritis, Parsonage-Turner syndrome, has an acute presentation following a painful episode involving the whole arm. There is significant history of a fall in this case.
Correct Answer: Postganglionic C 5, C 6
1) Magnetic resonance imaging (MRI)
3) Repeat electromyelogram (EMG) after 4 weeks
2) Computed tomography (CT) scan of the neck
5) Careful neurological examination
4) Somatosensory evoked potential (SSEP)
Computed tomography scan of the cervical spine will not show the pseudomeningoceles nor provide any information on brachial plexus. Computed tomography may be needed in case of a suspected neck injury but does not form part of a brachial plexus work up.
Correct Answer: Computed tomography (CT) scan of the neck
1507. (2360) Q4-2818:
A 15-year-old boy presented with inability to elevate his right shoulder and flex his elbow. He sustained a fall from an all-terrain vehicle 8 weeks ago. He landed on the right shoulder and twisted his neck. Radiographs of the skull, chest, cervical and thoracic spine, and shoulder were normal. There was no loss of consciousness, chest pain, or breathing difficulties. The patient was observed in the hospital until stable and referred for follow-up in the hand clinic at 4 weeks. An electromyelogram (EMG) was scheduled. Clinical examination revealed weakness of deltoid, supraspinatus, infraspinatus, teres minor, biceps, brachialis, brachioradialis, and extensor carpi radialis longus. The remainder of his forearm musculature was preserved and he could grasp, release, and pinch. Sensations were decreased along the distribution of the axillary nerve. There was 3 cm wasting of his arm and 2 cm of the forearm. Tinelâs sign is positive around the clavicle. Hornerâs signs are absent and his arm lies against the body. The EMG report showed fibrillation potentials in the weak muscles. The patient can now flex his elbow. When asked to demonstrate, he flexes his wrist and pronates his forearm to swing his elbow into flexion.
The plan of management in this patient 5 months postinjury with no clinical improvement should be:
1) Neurotization
3) Continued observation
2) Exploration and nerve grafting
5) Shoulder arthrodesis
4) Tendon transfers
Neurotization is appropriate in preganglionic lesions. If at 6 months a patient shows no evidence of recovery, it is time for plexus exploration. Further observation will not change the picture. Tendon transfers are reconstructive procedures, which are done at a later stage.
Correct Answer: Exploration and nerve grafting
1) Absence of biceps function at 3 months
3) Presence of trick movements
2) Absence of biceps function with return of extensor carpi radialis longus (ECRL) power at 4 months
5) Weakness of the supraspinatus
4) Subluxation of humeral head on radiographs
An important indication for early exploration is the recovery of a distally supplied muscle, ECRLâC 6, in the absence of a proximally supplied muscle, bicepsâC 5. Trick movements are adaptive movements employed by the patient by recruiting other muscles, for example, the use of flexor-pronator as elbow flexors in this patient. Bony deformity is a late sequelae and biceps recovery at 3 months is important in obstetric brachial palsy.
Correct Answer: Absence of biceps function with return of extensor carpi radialis longus (ECRL) power at 4 months
1509. (2362) Q4-2820:
A 15-year-old boy presented with inability to elevate his right shoulder and flex his elbow. He sustained a fall from an all-terrain vehicle eight weeks prior. He landed on the right shoulder and twisted his neck. Radiographs of the skull, chest, cervical and thoracic spine, and shoulder were normal. There was no loss of consciousness, chest pain, or breathing difficulties. The patient was observed in the hospital until stable and referred for follow-up in the hand clinic at 4 weeks. An electromyelogram (EMG) was scheduled. Clinical examination revealed weakness of deltoid, supraspinatus, infraspinatus, teres minor, biceps, brachialis, brachioradialis, and extensor carpi radialis longus. The remainder of his forearm musculature was preserved and he could grasp, release, and pinch. Sensations were decreased along the distribution of the axillary nerve. There was 3-cm wasting of his arm and 2 cm of the forearm. Tinelâs sign is positive around the clavicle. Hornerâs signs are absent and his arm lies against the body. The EMG report showed fibrillation potentials in the weak muscles. The patient can now flex his elbow. When asked to demonstrate, he flexes his wrist and pronates his forearm to swing his elbow into flexion.
The most important function that needs to be restored in this patient is:
1) Shoulder abduction
3) Elbow flexion
2) Shoulder elevation
5) Elbow extension
4) Wrist extension
Elbow flexion is central to management of brachial plexus management because it serves the most important function of feeding. Correct Answer: Elbow flexion
1) Erbâs palsy
3) Cerebrovascular accident
2) Klumpkeâs palsy
5) Syringomyelia
4) Ulnar and median combined nerve injury
This is a case of obstetric brachial plexus injury involving the C 8, T1 roots (Klumpkeâs palsy). Erbâs palsy involves upper roots only. Combined nerve injuries can present in a similar fashion, however low ulnar and median nerve lesions will not have weakness of the flexor digitorum profundus and flexor digitorum sublimis.
History of a large baby, shoulder dystocia, and clavicle fracture point to difficult labor. The most common type of brachial plexus injury related to birth is Erbâs palsy, which is usually associated with a breech presentation. Isolated Klumpkeâs palsy is quite rare and the involvement of C 8 and T1 usually occurs as part of global plexus injury.
Correct Answer: Klumpkeâs palsy
1511. (2364) Q4-2822:
An 18-month-old boy presents with a clawing deformity of the right hand. He was born full term after a difficult delivery complicated by shoulder dystocia. He weighed 9.5 lbs at birth. The patient had a brief episode of apnea with an APGAR score of 5 at birth and needed resuscitation and admission to the neonatal intensive care unit. A tender bump was noted on the patientâs right clavicle, which was diagnosed as clavicle fracture. A week later, the patient could not flex the fingers of his right hand. The neonatologist informed the parents that the fracture was managed conservatively and the absence of finger flexion was due to fracture and would recover. However, recovery can be prolonged and may take up to two years. The patient has grown and his immunization is complete. His right hand has extension at all the metacarpal joints of the fingers while the proximal interphalangeal and distal interphalangeal joints are flexed. The thumb is in an adducted position, and it is difficult to passively bring the thumb to full abduction. There is obvious wasting of the hand and forearm. The patient moves the arm well with no abnormalities noticed at the shoulder, elbow, and wrist. Radiograph of the chest shows a healed clavicle fracture with no evidence of diaphragmatic paralysis. There is no evidence of Hornerâs syndrome and the grasp reflex is absent.
The level of the lesion in this patient is:
1) Preganglionic lesion
3) Lateral cord
2) Postganglionic lesion
5) Upper trunk
4) Posterior cord
It is difficult to clinically differentiate between a pre- and postganglionic lesion of C 8, T1 in a child. Absence of Hornerâs syndrome and hemi-diaphragmatic palsy in this case indicates that this is not a preganglionic lesion. The ability of the patient to hold his head suggests that the paravertebral muscles are functional, as is true in postganglionic lesions.
Correct Answer: Postganglionic lesion
1) Neurotization
3) Tendon transfers
2) Exploration and nerve grafting
5) Vascularized nerve grafting
4) Neurolysis
Neurotization is done for preganglionic lesions and has not been shown to produce successful results for lower root involvement. At 18 months, exploration and nerve grafting must be carried out. Neurolysis is reserved for cases in which recovery is partial or plateaus. Tendon transfers in children less than 3 years old do not work as well. Younger children do not cooperate well in rehabilitation. It is also difficult to decide upon the functioning motors for transfer.
Correct Answer: Exploration and nerve grafting
1513. (2366) Q4-2825:
An 18-month-old boy presents with a clawing deformity of the right hand. He was born full term after a difficult delivery complicated by shoulder dystocia. He weighed 9.5 lbs at birth. The patient had a brief episode of apnea with an APGAR score of 5 at birth and needed resuscitation and admission to the neonatal intensive care unit. A tender bump was noted on the patientâs right clavicle, which was diagnosed as clavicle fracture. A week later, the patient could not flex the fingers of his right hand. The neonatologist informed the parents that the fracture was managed conservatively and the absence of finger flexion was due to fracture and would recover. However, recovery can be prolonged and may take up to two years. The patient has grown and his immunization is complete. His right hand has extension at all the metacarpal joints of the fingers while the proximal interphalangeal and distal interphalangeal joints are flexed. The thumb is in an adducted position, and it is difficult to passively bring the thumb to full abduction. There is obvious wasting of the hand and forearm. The patient moves the arm well with no abnormalities noticed at the shoulder, elbow, and wrist. Radiograph of the chest shows a healed clavicle fracture with no evidence of diaphragmatic paralysis. There is no evidence of Hornerâs syndrome and the grasp reflex is absent.
Reconstructive surgery includes all of the following except:
1) Thumb opposition
3) Thumb adduction
2) Widening of first web space
5) Thumb capsulodesis
4) Thumb metaphalangeal (MP) fusion
This patient has developed contractures of first web space, which will not respond to passive stretching. Fusion of the MP joint is unneccesary, as tendon transfers will provide lateral and tip pinch as well as opposition.
Correct Answer: Thumb metaphalangeal (MP) fusion
1) Volar proximal interphalangeal (PIP) joint dislocation and swan-neck deformity
3) Volar PIP joint dislocation and boutonniere deformity
2) Dorsal PIP joint dislocation and swan-neck deformity
5) Dorsal PIP joint dislocation and mallet finger deformity
4) Dorsal PIP joint dislocation and boutonniere deformity
Three types of PIP joint dislocations are identified: volar, dorsal, and central depression. Volar PIP joint dislocations result in avulsion of the dorsal fragment of the base of the middle phalanx, which represents the central tendonâs attachment. If displacement persists, than a boutonniere deformity may result. Volar avulsion fractures of the PIP joint are often due to a hyperextension injury at the attachment of the volar plate.Correct Answer: Volar PIP joint dislocation and boutonniere deformity
1515. (2368) Q4-2827:
At what degree of flexion is ulnar collateral ligament injury tested:
1) 0º of metacarpophalangeal (MCP) joint flexion
3) 60º of MCP joint flexion
2) 30º of MCP joint flexion
5) 120º of MCP joint flexion
4) 90º of MCP joint flexion
At 30º of MCP joint flexion, the ulnar collateral ligament is isolated from the volar plate.Correct Answer: 30º of MCP joint flexion
1516. (2369) Q4-2828:
Which of the following structures are found within the first dorsal compartment:
1) Abductor pollicis longus and extensor indicis
3) Abductor pollicis longus and extensor pollicis brevis
2) Abductor pollicis longus and extensor pollicis longus
5) Extensor carpi radialis longus and extensor pollicis brevis
4) Abductor pollicis brevis and extensor pollicis longus
The first dorsal compartment encompasses the abductor pollicis longus and extensor pollicis brevis. Multiple slips of abductor pollicus brevis may be present, which is important in de Quervainâs release.Correct Answer: Abductor pollicis longus and extensor pollicis brevis
1517. (2370) Q4-2829:
A 28-year-old man fell off his bike and sustained a fall onto his outstretched hand. He experiences thumb and index finger numbness. Attempts at reduction of his grade I open extra-articular distal radius fracture are unsuccessful. The next appropriate step of management is:
1) Incision and drainage, splint in functional position, and bone grafting
3) Incision and drainage, open reduction with internal fixation
2) Incision and drainage, carpal tunnel release, and splint in functional position
5) Incision and drainage, open reduction with internal fixation, carpal tunnel release, and bone grafting
4) Incision and drainage, open reduction with internal fixation, and carpal tunnel release
A patient with this injury represents a high-energy fracture in a high demand individual. The patient will require incision and drainage of his open wound, open reduction with internal fixation, and carpal tunnel release. Bone grafting would not be appropriate in a patient with open fracture.Correct Answer: Incision and drainage, open reduction with internal fixation, and carpal tunnel release
1) Thrombocytopenia absent radii
3) Holt-Oram syndrome
2) Fanconi anemia
5) Cardiac anomalies
4) Larsen syndrome
Patients with thrombocytopenia absent radii, Fanconi anemia, Holt-Oram syndrome, and cardiac anomolies all are associated with radial deficiency. Larsen syndrome is associated with multiple, larger joint dislocation.Correct Answer: Larsen syndrome
1519. (2372) Q4-2831:
Which of the following is the most common carpal coalition in the hand:
1) Lunotriquetral
3) Capitohamate
2) Scapholunate
5) Capitolunate
4) Radioscaphoid
Lunotriquetral coalition has a 1.6% prevalence in the general population. The second most common coalition is the capitohamate. Incomplete coalition is treated by arthrodesis of the lunotriquetral joint.Correct Answer: Lunotriquetral
1520. (2373) Q4-2832:
A 6-year-old boy presents with a Salter-Harris II distal radius fracture 3 weeks after injury. He is nontender and neurologically intact. On radiographs, he has a 35º dorsal angulation. The appropriate course of treatment is:
1) Observe, cast, follow until healed
3) Close reduction and casting
2) Observe, cast, follow with serial radiographs for at least 2 years
5) Open reduction
4) Close reduction and pins
For a patient with delayed presentation of a distal radius fracture, appropriate management includes casting and observation for at least 2 years to assess physeal damage and remodeling. The patient may require osteotomy if remodeling does not occur.Correct Answer: Observe, cast, follow with serial radiographs for at least 2 years
1521. (2374) Q4-2833:
The oblique retinacular ligament connects with what two structures:
1) Flexor tendon to lateral extensor tendon
3) Flexor tendon sheath to lateral extensor tendon
2) Flexor tendon to central slip
5) Flexor tendon sheath to head of middle phalanx
4) Flexor tendon sheath to central slip
Landsmeer (oblique retinacular ligament) runs from the flexor tendon sheath of the proximal phalanx to the lateral extensor tendon as they insert onto the base of the proximal phalanx. A stay or retaining ligament maintains centralization of the extensor tendons.Correct Answer: Flexor tendon sheath to lateral extensor tendon
her elbow is at 90º of flexion and pulp-to-pulp contact on key pinch. The most likely diagnosis is:
1) Carpal tunnel syndrome
3) Posterior interosseous nerve syndrome
2) Anterior interosseous nerve syndrome
5) Martin-Gruber connection
4) Cubital tunnel syndrome
Anterior interosseous nerve syndrome is due to compression of the anterior interosseous nerve (AIN) in the forearm by lacterus fibrosis, flexor digitorum superficialis, or pronator teres. The AIN innervates the pronator quadratus, flexor digitorum profundus (FDP) to the index finger and the flexor pollicis longus (FPL). Anatomy variation exists where the AIN may innervate part of the flexor digitorum superficialis. In this patient, she has decreased pronation at 90º flexion, which relaxes the humeral attachment of the pronator from the pronator quadrus weakness. She also has pulp-to-pulp contract due to weakness of the FPL and FDP to the index finger.Correct Answer: Anterior interosseous nerve syndrome
1523. (2376) Q4-2835:
Indications for operative treatment in an acute elbow dislocation include:
1) Instability to valgus stress
3) Radial head fracture involving 30% of the radial head
2) Recurrent dislocation with extension past 50º
5) Ulnar nerve parathesias
4) Osteochondral lesions
Recurrent dislocations with extension past 50° represent a significant injury to the elbow and require a stabilization period. Instability to valgus stress represents injury to the anterior band of the medial collateral ligament of the elbow and will heal with protected motion. The majority of radial head fractures (Mason type I and II) that are less than 30º of the radial head and less than 30º angulation heal with good functional results. Most dislocations will have osteochondral lesions. Ulnar nerve parathesias can be associated with dislocations but is not an indication for operative fixation.Correct Answer: Recurrent dislocation with extension past 50º
1524. (2377) Q4-2836:
When performing open reduction and internal fixation of radial neck fractures, the plate should be placed:
1) In the "nonarticular safe-zone" comprising 120º of the 360º radial head circumference
3) Forearm in supination with plate anterior
2) Forearm in pronation with plate posterior
5) Forearm in neutral with plate anterior
4) Forearm in supination with plate posterior
The "nonarticular safe-zone" comprising only 90º of the radial head circumference is achieved by placing the plate posterior with the arm in supination.Correct Answer: Forearm in supination with plate posterior
1525. (2378) Q4-2837:
Heterotopic ossification after elbow dislocations is not associated with which of the following:
1) Delay surgical intervention
3) Aggressive passive range of motion after dislocation
2) Closed head injury
5) Concomitant proximal humeral fracture
4) Extensive surgical dissection
Heterotopic ossification is commonly associated with delay of surgical intervention, closed head injury, aggressive passive range of motion after dislocation, and extensive surgical dissection. Radiographic evidence of heterotopic ossification is present in 75% of patients with elbow dislocations but only 5% of these are clinically significant.Correct Answer: Concomitant proximal humeral fracture
What is the order of joint destruction in a patient with scapholunate disassociation:
1) Radial styloid, proximal radioscaphoid, radiolunate, midcarpal
3) Proximal radioscaphoid, midcarpal, radiolunate
2) Radial styloid, proximal radioscaphoid, radiolunate
5) Radial styloid, proximal radioscaphoid, midcarpal
4) Proximal radioscaphoid, radial styloid, midcarpal, radiolunate
Patients with scapholunate disassociation can develop a scapholunate advanced collapsed wrist. The progression is from the radial styloid to proximal radioscaphoid, to midcarpal (capitolunate). The lunate is extended and unloaded due to its concentric design, which results in preservation of the radiolunate.Correct Answer: Radial styloid, proximal radioscaphoid, midcarpal
1527. (2380) Q4-2839:
Which of the following is not characteristic of Dupuytrenâs disease:
1) Autosomal dominant trait
3) Higher prevalence in men
2) Irish and Scottish decent
5) Predictable progression of disease
4) Ring and small finger involvement first
Dupuytrenâs disease is characteristically unpredictable in its clinical progression. It may spontaneously resolve or quickly progress to advanced disease.Correct Answer: Predictable progression of disease
1528. (2381) Q4-2840:
Operative indications for Dupuytrenâs contracture include:
1) Metacarpophalangeal joint contraction of more than 25º to 30º
3) Palpable cords in the palm
2) Proximal interphalangeal joint contracture of 30º or more
5) Painful palmar nodule
4) Decreased light touch sensation to affected digits
As a general guideline the "table test" is used as an indication for operative intervention. If the patient cannot lay his/her hand flat onto a table, the disease has usually progressed to the point where surgery is required. A metacarpophalangeal joint contracture of 30º to 40º or a proximal interphalangeal joint contracture of 30º or more is an indication for surgery.Correct Answer: Proximal interphalangeal joint contracture of 30º or more
1529. (2382) Q4-2841:
Favorable indications for attempted replantation include:
1) Amputation of the thumb
3) Crush injuries to the distal forearm
2) Warm ischemia time of less than 16 hours
5) Sharp amputation proximal to the elbow
4) Cold ischemia time of less than 20 hours
Favorable indications for replantation include thumb amputations because of the functional importance of the thumb. Warm ischemias less than 8 hours or cold ischemia time less than 16 hours are more favorable for replantation.Correct Answer: Amputation of the thumb
Injuries to the central articular disk portion of the triangular fibrocartilage complex are related to all of the following except:
1) Age
3) Ulnocarpal impingement
2) Positive ulnar variance
5) Avulsion injuries from the dorsal ligamentous attachments
4) Scaphoid nonunion
Scaphoid nonunion is not related to central triangular fibrocartilage complex injuries. A positive ulnar variance is most strongly associated with triangular fibrocartilage complex central disk injuries.Correct Answer: Scaphoid nonunion
1531. (2384) Q4-2843:
A patient reports that he felt a pop and immediate pain over the MP joint of his finger. Examination reveals tenderness on the dorsum of the joint and subluxation of the extensor tendon. Which of the following is the most common defect:
1) Central slip
3) Triangular ligament
2) Lateral bands
5) Extensor tendon
4) Sagittal fibers
Tears of the sagittal fibers of the dorsal aponeurosis result in subluxation of the extensor tendon. This usually occurs on the long finger with subluxation to the ulnar side. Treatment for acute injuries requires immobilization of the metacarpophalangeal joint in extension for 6 weeks. Treatment for chronic injuries includes repair of the torn radial sagittal fibers.Correct Answer: Sagittal fibers
1532. (2385) Q4-2844:
All of the following transfers may be used to improve function in a patient who has had radial nerve paralysis longer than 6 months, except:
1) Pronator to extensor carpi radialis brevis
3) Flexor digitorum superficialis of the ring finger to digital extensors
2) Flexor carpi radialis extensors
5) Flexor palmaris longus to extensor pollicis longus
4) Flexor digitorum superficialis of the ring finger to brachioradialis
Radial nerve paralysis is a common injury, and many patients recover after repair. Tendon transfers should be delayed until sufficient time for reinnervation has passed. Pronator to extensor carpi radialis brevis can be performed at time of nerve repair to provide wrist extension and grasp during period of nerve recovery. Transfers for radial nerve palsy need to address wrist extension, thumb extension, and finger extenstion. All of the above transfer would provide these functions except a transfer to the brachioradialis.Correct Answer: Flexor digitorum superficialis of the ring finger to brachioradialis
Slide 1
A 24-year-old man presents with a short forearm and a hand deformity. The patient is otherwise healthy with no other congenital defects. The clinical appearance of his forearm is shown (Slide). Your diagnosis is:
1) Ulnar club hand
3) Postaxial longitudinal deficiency
2) Preaxial longitudinal deficiency
5) Hypoplastic hand syndrome
4) Thumb aplasia
This is a classic appearance of a radial club hand, which is often referred to as preaxial longitudinal deficiency. Ulnar club hand and postaxial longitudinal deficiency are synonymous.Correct Answer: Preaxial longitudinal deficiency
1534. (2387) Q4-2846:
|
Slide 1
A 24-year-old man presents with a short forearm and a hand deformity. The patient is otherwise healthy with no other congenital defects. The clinical appearance of his forearm is shown (Slide). The patient has an elbow flexion contracture of 70° and desires lengthening. Which of the following statements is not true regarding lengthening:
1) Nerve palsies may occur during lengthening.
3) Lengthening usually equalizes limb length.
2) Lengthening must be done gradually.
5) Lengthening leads to recurrence of the deformity.
4) Lengthening helps improve function by extending the reach.
In most cases of radial club hand, excluding a hypoplastic radius, full correction cannot be achieved.Correct Answer: Lengthening usually equalizes limb length.
Slide 1
A 24-year-old man presents with a short forearm and a hand deformity. The patient is otherwise healthy with no other congenital defects. The clinical appearance of his forearm is shown (Slide). The potential complications of lengthening are discussed, and the patient is advised against it. However, the elbow flexion contracture is corrected by gradual distraction. One year postoperatively, the patient has attained a 30° correction of the flexion deformity, which remains mobile. Now, he desires that his wrist deformity be corrected. The procedure of choice is:
1) Arthrodesis
3) Centralization
2) Radialization
5) Tendon transfers
4) Proximal row carpectomy
Wrist arthrodesis is the best solution for this patient and his recurrent deformity because it provides a stable platform for grasp.Correct Answer: Arthrodesis
1536. (2389) Q4-2848:
|
Slide 1
A 24-year-old man presents with a short forearm and a hand deformity. The patient is otherwise healthy with no other congenital defects. The clinical appearance of his forearm is shown (Slide). Although the patient has a thumb, it is in an abnormal position. Any attempt to make his thumb more functional will be influenced by:
1) Index finger camptodactyly
3) Stiff fingers
2) Presence of a side-to-side finger grip
5) Radial bowing
4) Recurrence of radial club hand
The pattern of usage of the hand is established in the brain by 2 to 3 years of age. Although pollicization has been performed in adolescents, patients continue to prefer a scissor pinch. At 24 years of age, this pattern will be well established. The patient can be coaxed to use his thumb, but it will not be involuntary and automatic.Correct Answer: Presence of a side-to-side finger grip
Slide 1
A radial club hand is the result of an insult during which phase of the gestation period:
1) Weeks 1 to 4
3) Weeks 8 to 12
2) Weeks 4 to 7
5) Anytime during gestation
4) Weeks 12 to 16
A radial club hand is the result of an insult during weeks 4 to 7 of gestation.Correct Answer: Weeks 4 to 7
1538. (2391) Q4-2850:
A 15-day-old boy presents with deformity of the right hand. The boy was delivered prematurely and underwent an urgent arterial switch for transposition of great vessels. The patient is in stable condition. He has a radial club hand, and because the radial head cannot be palpated, total absence of radius is suspected. The thumb is absent and the index finger has camptodactyly. The forearm is short compared to the left side, and the patient flexes his elbow upon stimulation. Spontaneous finger motion is also present. A thorough physical examination is performed and a set of investigations is ordered. The results are as follows: complete blood count 10,000 mcu/L; platelet 254 254×103 mcu/L; neutophils 50%; Hb 14.2 mg/dL; lymphocytes 40%; Hct 45; and monocytes 10%. No renal abnormalities were noted on ultrasonogram of the abdomen. A radiograph of the spine is normal.
Diagnosis is:
1) Vertebral defects, anal atresia, tracheoesophageal fistula with esophageal atresia, and radial and renal anomalies (VATER)
3) Thrombocytopenia absent radii (TAR) syndrome
2) Abnormalities of vertebrae, anus, cardiovascular tree, trachea, esophagus, renal system, and limb buds (VACTERL)
5) Fanconiâs anemia
4) Holt-Oram syndrome
The patient has a radial club hand with a cardiac defect. Because the spine radiograph is normal, the diagnosis cannot be VATER or VACTERL anomaly as both involve vertebrae. Blood work up is normal, making this diagnosis Holt-Oram syndrome.Correct Answer: Holt-Oram syndrome
1539. (2392) Q4-2851:
The principal abnormality associated with Holt-Oram syndrome is:
1) Platelet deficiency
3) Pancytopenia
2) Cardiac defects
5) Vertebral defects
4) Malignancy
Holt-Oram syndrome is associated with cardiac defects. The most common defect is aldosterone secretion.Correct Answer: Cardiac defects
The hereditary pattern for Holt-Oram syndrome is:
1) Autosomal recessive
3) Sex-linked recessive
2) Autosomal dominant
5) Sporadic
4) Sex-linked dominant
Holt-Oram syndrome is inherited in an autosomal dominant manner.Correct Answer: Autosomal dominant
1541. (2394) Q4-2853:
A 15-day-old boy presents with deformity of the right hand. The boy was delivered prematurely and underwent an urgent arterial switch for transposition of great vessels. The patient is in stable condition. He has a radial club hand, and because the radial head cannot be palpated, total absence of radius is suspected. The thumb is absent and the index finger has camptodactyly. The forearm is short compared to the left side, and the patient flexes his elbow upon stimulation. Spontaneous finger motion is also present. A thorough physical examination is performed and a set of investigations is ordered. The results are as follows: complete blood count 10,000 mcu/L; 254×103 mcu/L; neutophils 50%; Hb 14.2 mg/dL; lymphocytes 40%; Hct 45; and monocytes 10%. No renal abnormalities were noted on ultrasonogram of the abdomen. A radiograph of the spine is normal.
The next step in the management of the radial club hand is:
1) Stretching
3) Radialization
2) Soft-tissue distraction
5) Pollicization
4) Centralization
For the first 6 to 9 months, parents and therapists perform passive stretching. Serial casting and splinting may also be used. Sometimes, preoperative soft-tissue distraction is performed, usually before a wrist stabilization procedure.Correct Answer: Stretching
1542. (2395) Q4-2854:
A 15-day-old boy presents with deformity of the right hand. The boy was delivered prematurely and underwent an urgent arterial switch for transposition of great vessels. The patient is in stable condition. He has a radial club hand, and because the radial head cannot be palpated, total absence of radius is suspected. The thumb is absent and the index finger has camptodactyly. The forearm is short compared to the left side, and the patient flexes his elbow upon stimulation. Spontaneous finger motion is also present. A thorough physical examination is performed and a set of investigations is ordered. The results are as follows: complete blood count 10,000 mcu/L; platelet 254×103 mcu/L; neutophils 50%; Hb 14.2 mg/dL; lymphocytes 40%; Hct 45; and monocytes 10%. No renal abnormalities were noted on ultrasonogram of the abdomen. A radiograph of the spine is normal.
Centralization will be performed on the patient. All of the following statements are true about centralization except:
1) It is necessary to make a notch in the carpus when performing centralization.
3) Preoperative soft tissue distraction can be useful.
2) The forearm must be aligned with the second metacarpal.
5) Ulnocarpal fusion is a known outcome.
4) Transfer of tendons from the radial to ulnar side provides additional stability.
In a centralization procedure, the forearm is aligned with the third metacarpal, not the second.Correct Answer: The forearm must be aligned with the second metacarpal.
A 15-day-old boy presents with deformity of the right hand. The boy was delivered prematurely and underwent an urgent arterial switch for transposition of great vessels. The patient is in stable condition. He has a radial club hand, and because the radial head cannot be palpated, total absence of radius is suspected. The thumb is absent and the index finger has camptodactyly. The forearm is short compared to the left side, and the patient flexes his elbow upon stimulation. Spontaneous finger motion is also present. A thorough physical examination is performed and a set of investigations is ordered. The results are as follows: complete blood count 10,000 mcu/L; 254Ã103 mcu/L; neutophils 50%; Hb 14.2 mg/dL; lymphocytes 40%; Hct 45; and monocytes 10%. No renal abnormalities were noted on ultrasonogram of the abdomen. A radiograph of the spine is normal.
When the patient is 10 years old, he is not satisfied with the length of his forearm and wishes to lengthen it. Which of the following is not a satisfactory recommendation:
1) Acute lengthening with bone graft
3) Hybrid frame and distraction using the Ilizarov method
2) Circular ring fixator and gradual distraction
5) Lengthening
4) External frame and distraction using De Bastianiâs principles
Acute lengthening is done for small defects and, if performed in this patient, may result in severe neurovascular compromise.Correct Answer: Acute lengthening with bone graft
1544. (2397) Q4-2856:
Which of the following conditions is present in patients with radial club hand but not in patients with ulnar club hand:
1) Thumb hypoplasia
3) Short forearm
2) Thumb aplasia
5) Bowing of the forearm
4) Renal malformations
Ulnar club hand differs from radial club hand in that cardiopulmonary, hematopoeitic, gastrointestinal, and genitourinary anomalies are uncommon.
Correct Answer: Renal malformations
1545. (2398) Q4-2857:
All of the following developmental anomalies are associated with ulnar club hand except:
1) Atrial septal defects
3) Fibular agenesis
2) Proximal focal femoral deficiencies
5) Radial ray defects
4) Mental retardation
Atrial septal defects are developmental abnormalities present in patients with radial club hand or Holt-Oram syndrome. Correct Answer: Atrial septal defects
Which of the following syndromes is associated with ulnar club hand:
1) Vertebral defects, anal atresia, tracheoesophageal fistula with esophageal atresia, and radial and renal anomalies (VATER)
3) Holt-Oram syndrome
2) Abnormalities of vertebrae, anus, cardiovascular tree, trachea, esophagus, renal system, and limb buds (VACTERL)
5) Femur-fibular-ulnar syndrome
4) Thrombocytopenia absent radii (TAR) syndrome
VATER, VACTERL, Holt-Oram syndrome, and TAR syndrome are associated with radial club hand. Femur-fibular-ulnar syndrome is characterized by proximal femoral focal deficiency, fibular agenesis, and ulnar ray defects.
Correct Answer: Femur-fibular-ulnar syndrome
1547. (2400) Q4-2859:
Which of the following areas is not involved in ulnar club hand:
1) Thumb
3) Shoulder
2) Elbow
5) Vertebra
4) Femur
Vertebrae are usually not involved in ulnar club hand. Correct Answer: Vertebra
1548. (2401) Q4-2860:
All of the following are true statements regarding elbow involvement in ulnar club hand except:
1) Fifty percent of patients have radial head dislocation.
3) Elbow instability worsens with the severity on involvement.
2) Nearly 50% of aplasia patients have radiohumeral synostosis.
5) The elbow is usually normal in all hypoplastic patients.
4) The anlage causes radial head dislocation or subluxation.
Elbow instability does not correspond with severity of involvement. Fifty percent of patients with total aplasia have radiohumeral synostosis, which provides adequate stability.
Correct Answer: Elbow instability worsens with the severity on involvement.
1549. (2402) Q4-2861:
All of the following statements are true regarding the carpal bones in patients with ulnar club hand except:
1) Involvement of carpus is severe in type III.
3) Carpal coalition is present in approximately 25% of patients.
2) The pisiform is the most common missing carpus.
5) The extent of ulnar deformity does not correlate with deformities in the hand.
4) Making a notch in the carpus provides stability at the wrist joint.
A notch is often created in centralization procedures for radial club hand. Wrist stabilization procedures are not performed for ulnar club hand.
Correct Answer: Making a notch in the carpus provides stability at the wrist joint.
All of the following anomalies are present in patients with ulnar club hand except:
1) Phocomelia
3) Humeral aplasia
2) Transverse arrest
5) Vertebral dysplasia
4) Humeral hypoplasia
Vertebral anomalies are not common in patients with ulnar club hand. Correct Answer: Vertebral dysplasia
1551. (2404) Q4-2863:
|
Slide 1
A 1-year-old boy was born full-term and pregnancy was uneventful. However, the parents noticed deformity of the patientâs forearm, which progressed with growth. The parents consulted a pediatric orthopaedic surgeon 4 months prior and were advised to observe the growth. Multiple investigations in the form of two-dimensional echogram, abdomen ultrasonography, radiographs of the spine, and complete blood work did not reveal any abnormalities. No genetic or syndromic abnormality was reported. A radiograph taken 4 months prior is shown.
1) Postaxial longitudinal deficiency
3) Ulnar agenesis
2) Preaxial longitudinal deficiency
5) Cleft hand
4) Radial club hand
The ulnar deficiency is longitudinal and the ulna is considered a postaxial bone. Ulnar agenesis means absence while radial club hand is a pre-axial longitudinal deficiency and cleft hand is a central deficiency.
Correct Answer: Postaxial longitudinal deficiency
1552. (2689) Q4-3182:
A 45-year-old man presents with marked lateral elbow pain. He says that the pain has been present for 3 weeks. He has no history of recent trauma. He is an avid tennis player, and he feels increased pain after playing tennis and when doing wrist extension exercises in the gym. His pain is maximally reproduced with resisted middle finger extension and with forearm supination with the elbow extended. Electromyography would confirm the diagnosis as:
1) Radial head fracture
3) Lateral ulnar collateral ligament tear
2) Lateral condyle fracture
5) Radial tunnel syndrome
4) Lateral epicondylitis
The clinical picture is similar to that of lateral epicondylitis. However, the maximal tenderness is slightly more distal, just beyond the radial head. Diagnosis may be confirmed using provocative maneuvers (resisted middle finger extension or forearm supination with the elbow extended) or with electromyography.Correct Answer: Radial tunnel syndrome
A 45-year-old man presents with marked lateral elbow pain. He says that the pain has been present for 3 weeks. He has no history of recent trauma. He is an avid tennis player, and he feels increased pain after playing tennis and when doing wrist extension exercises in the gym. His pain is maximally reproduced with resisted middle finger extension and with forearm supination with the elbow extended. Which of the following is the appropriate initial treatment:
1) Open reduction and internal fixation
3) Posterior interosseous nerve decompression
2) Arthroscopic ligament repair
5) Anti-inflammatory medication and a program of muscle strengthening
4) Release of the extensor carpi radialis brevis origin off the lateral epicondyle
Ninety to 95% of all patients with tennis elbow respond to nonoperative treatment, which should always be tried first. Treatment begins with a period of rest, ice, and nonsteroidal anti-inflammatory medications.Correct Answer: Anti-inflammatory medication and a program of muscle strengthening
1554. (2691) Q4-3184:
Lateral epicondylitis is associated with a tear in the fibers of which muscle:
1) Extensor carpi radialis brevis (ECRB)
3) Brachioradialis
2) Extensor carpi radialis longus (ECRL)
5) Anconeus
4) Supinator
Current consensus is that tennis elbow is associated with a strain or microtear of the ECRB origin, which lies beneath the ECRL.Correct Answer: Extensor carpi radialis brevis (ECRB)
1555. (2692) Q4-3185:
The gold standard for diagnosis of lateral epicondylitis is considered:
1) History and physical examination
3) Electromyography
2) Plain radiographs
5) Radionuclear bone scan
4) Magnetic resonance imaging
The clinical diagnosis of lateral epicondylitis is supported by specific provocative tests. The gold standard for diagnosis is the history and physical examination. Tenderness on examination is localized to the lateral epicondyle, which can radiate into the forearm; the area of maximum tenderness is approximately 2 mm to 5 mm distal and anterior to the midpoint of the lateral epicondyle. There is usually a history of overuse or of a repetitive activity. The pain is aggravated, with the elbow extended, by resisted wrist and finger extension or with passive finger and wrist flexion.Correct Answer: History and physical examination
1556. (4055) Q4-3186:
The amount of time that nonoperative management should be followed for lateral epicondylitis is closest to:
1) 1 day
3) 1 month
2) 1 week
5) 6 months or longer
4) 3 months
Ninety to 95% of all patients with tennis elbow respond to nonoperative treatment, and it remains the mainstay for treatment of lateral epicondylitis. Operative treatment may be indicated for debilitating pain in patients without other pathologic causes of pain for whom nonoperative treatment has failed after a reasonable length of time. This time period is usually a minimum of 6 to 12 months.Correct Answer: 6 months or longer
Extracorporeal shock wave therapy in the treatment of lateral epicondylitis in high-quality trials.
1) Is ineffective
3) Has not been tested
2) Is beneficial
5) Improves function
4) Improves short-term pain
Current studies have found no benefit of extracorporeal shock wave therapy in the treatment of lateral epicondylitis.Correct Answer: Is ineffective
1558. (2768) Q4-3266:
All of the following medications are indicated in the early treatment of frostbite injury except:
1) Ibuprofen
3) Antibiotic prophylaxis
2) Tetanus booster
5) Intravenous beta blocker
4) Narcotic pain medications
Peripheral beta-blockade has minimal effect on peripheral vasculature and is not typically used in patients with frostbite injury. Ibuprofen, tetanus booster, antibiotic prophylaxis, and appropriate pain medication are routinely used in the treatment of frostbite injuries.Correct Answer: Intravenous beta blocker
1559. (2769) Q4-3267:
Orthopedic sequelae of frostbite injury include all of the following except:
1) Joint contractures
3) Decreased risk of future frostbite injury
2) Localized osteoporosis
5) Cold intolerance
4) Punched-out subchondral bony lesions
People who have had previous frostbite injuries are at increased risk of thermal injury, whether cold or heat related. Joint contractures, localized osteoporosis, punched-out subchondral bony lesions, and cold intolerance are often present after frostbite injury.Correct Answer: Decreased risk of future frostbite injury
1560. (2770) Q4-3268:
Initial treatment of an acute frostbite injury should include:
1) Rapid rewarming in circulating 34° C to 36° C water
3) Rapid rewarming in steam
2) Rapid rewarming in circulating 40° C to 42° C water
5) Slow rewarming in room air
4) Slow rewarming with intermittent 50° C to 55° C water
Rapid rewarming in a 40° C to 42° C circulating water bath is the most effective early treatment of frostbite injury. Slow or fast rewarming in other temperatures and/or rewarming in air is not indicated.Correct Answer: Rapid rewarming in circulating 40° C to 42° C water
1) 32° C
3) 0° C
2) 10° C
5) â25° C
4) â10°C
Frostbite will generally not occur above â10° C, but the risk is significantly increased when the air temperature is below â25° C.Correct Answer: â25° C
1562. (2772) Q4-3270:
Superficial frostbite injuries result in:
1) Minimal tissue loss
3) Firm tissue
2) Significant tissue loss
5) Hemorrhagic blisters
4) Amputation
Superficial frostbite typically involves minimal tissue loss. Patients have pliable skin with sensation. Deep frostbite typically results in significant soft tissue loss with firm anesthetic tissue that forms hemorrhagic blisters.Correct Answer: Minimal tissue loss
1563. (2773) Q4-3271:
All of the following except increase the risk of frostbite injury.
1) Altitude higher than 17,000 feet
3) Increased humidity
2) History of smoking
5) Peripheral vascular disease
4) Prolonged exposure
High altitudes, prolonged exposure, and anything that would cause peripheral vasoconstriction increase the risk of frostbite injury. Humidity does not play a significant role in the development of frostbite injury.Correct Answer: Increased humidity
1564. (2774) Q4-3272:
Treatment for frostbite injury includes:
1) Limiting active motion of the frostbitten area
3) Stopping the rewarming process when there is pain secondary to reperfusion
2) Elevating the frostbitten extremity to reduce edema
5) Massaging the frostbitten area thoroughly to increase perfusion
4) Using dry heat
Treatment of frostbite includes rapid rewarming (even when reperfusion pain occurs), early active motion, elevation, and avoidance of dry heat that can dessicate tissues. Massaging the frostbitten area is not recommended because it may induce additional trauma via shearing forces.Correct Answer: Elevating the frostbitten extremity to reduce edema
Arthritis of the wrist is estimated to effect what percentage of the U.S. population:
-
Less than 1%
3) 5%
-
3%
5) More than 15%
4) 10%
Arthritis of the wrist is estimated to affect 5.3% of the U.S. population, based on radiographic assessments of 4,000 wrists.1 After having rheumatoid arthritis (RA) for 10 years, 90% of patients experience arthritis in their wrist joints.2
Correct Answer: 5%
1566. (2948) Q4-3451:
The accessory ulnar collateral ligament inserts on the:
1) Proper ulnar collateral ligament
3) Proximal phalanx
2) Lateral bands
5) Flexor sheath
4) Volar plate
The accessory ulnar collateral ligament inserts into the volar plate, whereas the proper collateral inserts into the base of the proximal phalanx.Correct Answer: Volar plate
1567. (3113) Q4-3623:
Which of the following nerves is not a primary articular nerve of the wrist:
1) Posterior interosseous nerve (PIN)
3) Palmar cutaneous branch of the median nerve
2) Lateral antebrachial cutaneous nerve
4) Articular branches from the median nerve
Fukumoto and colleagues have used Wykeâs definition to explain primary and accessory innervation of the wrist. Primary articular nerves consist of small nerves that pass to each joint as independent branches of adjacent peripheral nerves. There are three primary articular nerves: the PIN, the lateral antebrachial cutaneous nerve, and the articular branches from the ulnar nerve.
Accessory nerves originate from small, twig branches of intramuscular or cutaneous nerves that innervate the skin around the wrist joint. The accessory articular nerves have been identified as the anterior interosseous nerve (AIN), the palmar cutaneous branch of the median nerve, the deep and dorsal branches of the ulnar nerve, and the superficial branch of the radial nerve to the first intercarpal space.Correct Answer: Palmar cutaneous branch of the median nerve
1568. (3114) Q4-3624:
Which of the following nerves provides principal innervation to the central dorsal portion of the wrist:
1) Anterior interosseous nerve (AIN)
3) Dorsal branch of the ulnar nerve
2) Posterior interosseous nerve (PIN)
4) Lateral antebrachial cutaneous nerve
The PIN is found on the deep radial wall of the fourth dorsal compartment, 1.2 cm ulnar to Listerâs tubercle. As the PIN approaches the radiocarpal joint, it is covered in fascia and gives one branch to the radioscaphoid joint and three to four terminal branches to the intercarpal joints. The PIN is the principal innervation to the central dorsal portion of the wrist. The AIN innervates the radial volar lip of the distal radius. The dorsal branch of the ulnar nerve contributes to innervation of the triangular fibrocartilage complex. The lateral antebrachial cutaneous nerve innervates the thumb carpometacarpal joint and the scapho-trapezotrapezoid joint.Correct Answer: Posterior interosseous nerve (PIN)
What is the area of innervation of the anterior interosseous nerve (AIN):
1) Radial volar lip of the distal radius
3) Dorsal radiocarpal joint
2) Triangular fibrocartilage complex (TFCC)
4) Thumb carpometacarpal joint
The AIN is a branch of the median nerve. Its muscular innervations include the flexor pollicis longus, the radial half of the flexor digitorum profundus, and the pronator quadratus. The AIN terminates as a sensory branch to the volar radial surface of the distal radius. The TFCC is innervated by components of the ulnar nerve. The dorsal radiocarpal joint is innervated by the posterior interosseous nerve. The thumb carpometacarpal is innervated by the sensory branch of the radial nerve and the lateral antebrachial cutaneous nerve.Correct Answer: Radial volar lip of the distal radius
1570. (3116) Q4-3626:
When performing complete wrist denervation as described by Wilhem, what pain pathology did not have predictable results:
1) Scaphoid nonunion
3) Primary radiocarpal arthritis
2) Osteonecrosis of the scaphoid
4) Ulnar carpal arthritis
In 1983, Ekerot and colleagues reported his results in 48 patients. They used the technique described by Wilhelm but only denervated the radial side of the wrist for patients with scaphoid or lunate pathology. However, the entire wrist was denervated in patients with global degenerative wrist disease or wrist pain with an unknown etiology. Pain relief occurred in only 56% of the patients. They noted the best results occurred in patients with scaphoid nonunion, osteonecrosis of the lunate, and primary radiocarpal arthritis.Correct Answer: Ulnar carpal arthritis
1571. (3117) Q4-3627:
What two nerves are resected through a single dorsal incision for wrist denervation:
1) Superficial branch of the radial nerve and posterior interosseous nerve (PIN)
3) PIN and anterior interosseous nerve (AIN)
2) PIN and the dorsal cutaneous branch of the ulnar nerve
4) Superficial branch of the radial nerve and dorsal cutaneous branch of the ulnar nerve
Kupfer and colleagues presented a podium presentation of a single-incision approach to the resection of the PIN and AIN for denervation of the radial side of the wrist. Weinstein and Berger published their results in 2002 with a similar technique. They described a 2-cm long dorsal incision that was 3 to 5 cm proximal to the ulnar head. They then resected a 2-cm segment of the PIN and AIN. In their group of 20 patients, 85% were satisfied with their procedure after an average follow-up of 2.5 years. If failure were to occur, it occurred within the first year.Correct Answer: PIN and anterior interosseous nerve (AIN)
1572. (3118) Q4-3628:
What muscle is at risk for denervation when a single dorsal incision is used to denervate the radial side of the wrist:
1) Flexor pollicis longus
3) Flexor digitorum profundus
2) Extensor indicis
-
Pronator quadratus
The single dorsal incision approach to wrist denervation involves resection of the posterior interosseous nerve (PIN) and the anterior interosseous nerve (AIN). Distally, the PIN is purely sensory and does not give off motor branches in the vicinity of the wrist joint. The terminal portion of the AIN has both motor and sensory components. A majority of this is motor, and it innervates the pronator quadratus right up to the radiocarpal articulation. Resection of the AIN close to the radiocarpal joint has a high probability of denervating the pronator quadratus. The extensor indicis is usually the last motor branch of the PIN, but this terminal portion of this branch is more than 5 cm proximal from the distal radioulnar joint. The flexor pollicis longus is innervated by the AIN, but motor branches to this muscle are more proximal than branches to the pronator quadratus. The flexor digitorum profundus muscle is innervated by the AIN and ulnar nerve. However, the motor branches are more proximal than the incision for dorsal innervation.Correct Answer: Pronator quadratus
A 62-year-old man presents with weakness in finger extension in his right hand. He has had the weakness for 1 month but denies any significant traumatic event. The patient maintains an active lifestyle, including golf and tennis. He denies pain or numbness in his hand and is otherwise neurologically intact. Which of the following is the most likely diagnosis:
1) Thoracic outlet syndrome
3) Ulnar nerve palsy
2) Posterior interosseous nerve palsy
5) Radial nerve palsy
4) Carpal tunnel syndrome
Posterior interosseous nerve palsy is described as painless finger drop. This syndrome is commonly associated with trauma to the lateral elbow.Correct Answer: Posterior interosseous nerve palsy
1574. (3177) Q4-3996:
Which of the following are characteristic signs of PIN palsy:
1) Weakness in finger extension
3) Elbow tenderness
2) Pain in dorsum of hand
5) Weakness in finger extension, elbow tenderness, and pain in dorsum of hand
4) Weakness in finger extension, and elbow tenderness
Painless finger drop is characteristic of posterior interosseous nerve palsy. This syndrome may also involve elbow tenderness in the absence of other clinical findings. Pain in the dorsum of the hand is not associated with this condition because the posterior interosseous nerve contains no sensory component.Correct Answer: Weakness in finger extension, and elbow tenderness
1575. (3178) Q4-3997:
What is the most common site of posterior interosseous nerve entrapment:
1) The arcade of Frohse
3) The first cervical rib
2) The flexor retinaculum
5) Ligament of Struthers
4) In the spiral groove of the humerus
The most common site of posterior interosseous nerve entrapment is at the arcade of Frohse, which is a fibrotendinous ring found within the fibers of the supinator muscle as the posterior interosseous nerve originates from the radial nerve.Correct Answer: The arcade of Frohse
1576. (3179) Q4-3998:
Which of the following muscles is innervated by the posterior interosseous nerve:
1) Extensor carpi ulnaris
3) Extensor pollicis brevis and longus
2) Extensor digiti minimi
5) All of the above
4) Abductor pollicis longus
The posterior interosseous nerve innervates a number of muscles involved primarily in finger extension, including the extensor carpi ulnaris, extensor digitorum, extensor digiti minimi, extensor pollicis brevis and longus, abductor pollicis longus, and extensor indices.Correct Answer: All of the above
Posterior interosseous nerve palsy affects finger extension at the metacarpophalangeal and interphalangeal joints.
-
True
-
False
Only the metacarpophalangeal joints are involved in posterior interosseous nerve palsy, as the muscles of the interphalangeal joints are innervated by the ulnar and median nerves.Correct Answer: False
1578. (3181) Q4-4000:
A 53-year-old woman presents with bilateral hand numbness and tingling. Her right hand is more affected than her left. The numbness wakes her up at night and is relieved when she shakes her hand. In addition, the patient has had increasing difficulty with fine motor tasks, such as shirt buttoning, over the past 2 to 3 months. Upon close inspection, muscle atrophy is present at the base of her thumbs. Which of the following is the most likely diagnosis:
-
Thoracic outlet syndrome
-
-
Ulnar nerve palsy
-
Posterior interosseous nerve palsy
-
-
-
Radial nerve palsy
-
Carpal tunnel syndrome
This patient displays the classic signs and symptoms of bilateral carpal tunnel syndrome, which involves median nerve entrapment at the base of the palm. This entrapment leads to numbness and dysesthesias that are worse at night and upon exertion. Pain is typically relieved by shaking the hand. Furthermore, the median nerve innervates several muscles of the hands, and entrapment may lead to muscle atrophy.Correct Answer: Carpal tunnel syndrome
1579. (3182) Q4-4001:
All of the following muscles are innervated by the median nerve except:
1) The ulnar two lumbricals (lumbricals III and IV)
3) Abductor pollicis brevis
2) Opponens pollicis
5) Flexor digitorum profundus to the middle finger
4) Flexor pollicis brevis
Lumbricals 1 and 2 are innervated by the median nerve, in addition to the opponens pollicis brevis, abductor pollicis brevis, and flexor pollicis brevis.Correct Answer: The ulnar two lumbricals (lumbricals III and IV)
1580. (3183) Q4-4002:
Carpal tunnel syndrome is typically due to median nerve entrapment at the:
1) Arcade of Frohse
3) First cervical rib
2) Flexor retinaculum
5) Ligament of Struthers
4) Spiral groove of the humerus
The carpal tunnel is made by the flexor retinaculum, also known as the transverse carpal ligament.Correct Answer: Flexor retinaculum
1581. (3184) Q4-4003:
All of the following are true regarding the transverse carpal ligament except:
1) The transverse carpal ligament attaches medially to the pisiform and hook of hamate.
3) The palmaris longus tendon lies superficially to the transverse carpal ligament.
2) The transverse carpal ligament attaches laterally to the scaphoid and trapezium.
5) None of the above are true
4) All of the above are true
All of the above statements are true regarding the transverse carpal ligament.Correct Answer: All of the above are true
1582. (3185) Q4-4004:
All of the following structures pass through the carpal tunnel except:
1) Motor branch of the median nerve
3) Palmar cutaneous branch of median nerve
2) Tendon of the flexor pollicis longus
5) Tendon of the flexor digitorum sublimes
4) Tendon of the flexor digitorum profundus
The palmar cutaneous branch of the median nerve originates proximally to the carpal tunnel and travels superficial to the tunnel.Correct Answer: Palmar cutaneous branch of median nerve
1583. (3628) Q4-6515:
Dupuytrenâs contracture characteristically involves which part of the hand:
1) Ulnar side of the hand
3) Radial side of the hand
2) Thumb
5) Web space between the first and second metacarpals
4) Extensor tendons
Dupuytrenâs contracture most frequently involves the ring and small fingers. Although Dupuytrenâs cords at the thumb have been described, they are rare.Correct Answer: Ulnar side of the hand
1584. (3629) Q4-6516:
Which of the following cells are involved in Dupuytrenâs contracture:
1) Giant cells
3) Fibrocytes
2) Polymorphonuclear cells
5) Myocytes
4) Myofibroblasts
Gabbiani and Majno noted the abnormal appearance of fibroblasts found in patients with Dupuytrenâs contracture. They used the term myofibroblasts to describe these cells because they showed characteristics of fibrous and muscular tissue.Correct Answer: Myofibroblasts
1585. (3630) Q4-6517:
All of the following are contributory risk factors to the development of Dupuytrenâs contracture except:
1) Diabetes mellitus
3) Tobacco use
2) Hypertension
5) Human immunodeficiency virus (HIV)
4) Alcohol abuse
No association exists between high blood pressure and Dupuytrenâs disease, but all of the other listed conditions have been associated with Dupuytrenâs contracture. HIV has recently been described as a risk factor; patients with Dupuytrenâs contracture have been in advanced stages of HIV.Correct Answer: Hypertension
1) Injecting collagenase into the affected joint
3) Performing a subcutaneous wheal injection of collagenase
2) Injecting collagenase into the Dupuytrenâs cord
5) Applying collagenase to the Dupuytrenâs cords during surgery
4) Using a patch of collagenase on the skin
Clostridial collagenase works by breaking the collagen connections. The Dupuytrenâs cord is ruptured manually; surgery is not necessary.Correct Answer: Injecting collagenase into the Dupuytrenâs cord
1587. (3632) Q4-6519:
Dupuytrenâs cord tissue is characterized by what change from normal:
1) An increase in type II collagen
3) An increase of type III collagen
2) A decrease in type III collagen
5) Increased hyaluronidase
4) Abnormal collagen crosslinks
Compared to normal palmar fascia, the fibrous bands in Dupuytrenâs disease have an increased ratio of type III to type I collagen, and an overall increase in the amount of type III collagen.Correct Answer: An increase of type III collagen
1588. (3633) Q4-6520:
The strongest portion of the scapholunate interosseous ligament (SLIL) is the:
1) Proximal
3) Volar
2) Distal
5) Dorsal intercarpal
4) Dorsal
The SLIL is a c-shaped structure, which is thickest dorsally. The dorsal fibers have an average thickness of 3 mm and are composed of transversely oriented fibers, which afford the greatest resistance to translation between the scaphoid and the lunate, preventing the characteristic radiographic separation and flexion deformity of the scaphoid.Correct Answer: Dorsal
1589. (4063) Q4-6521:
A 29-year-old man with a remote history of wrist trauma and chronic pain presents with a palpable clunk on radio-ulnar deviation of the wrist. The most sensitive technique for identifying a scapholunate injury is:
1) Plain radiographs
3) Dynamic cineradiography
2) Magnetic resonance image (MRI)
5) Arthroscopy
4) Bone scan
Magnetic resonance imaging is commonly used among patients with concern for ligamentous injuries of the wrist, particularly in the presence of an abnormal physical exam when plain radiographs are normal. However, the sensitivity of MRI has been shown to be less than 40% in comparison with arthroscopy. Arthroscopy has become the gold standard for the diagnosis of ligamentous injuries to the wrist. A classification scheme has been proposed based on both radiocarpal and midcarpal arthroscopic findings.Correct Answer: Arthroscopy
1) Dorsal intercalated segment instability (DISI)
3) Terry Thomas sign
2) Volar intercalated segment instability (VISI)
5) Abnormal Gilulaâs arcs
4) Ring pole sign
On a lateral view of the wrist, when the lunate slips into a statically dorsiflexed position greater than 10°, the condition is defined as dorsal intercalated segmental instability (DISI). DISI deformity is also present when the scapholunate angle is greater than 60 degrees (45+/- 15 degrees is normal). The VISI deformity is seen on the lateral radiograph is characteristic of lunotriquetral dissociation. The other signs are seen on the anteroposterior projection.Correct Answer: Dorsal intercalated segment instability (DISI)
1591. (3635) Q4-6523:
A 40-year-old woman with radial sided wrist pain for the last 2 years presents to the clinic. Plain radiographs are normal. Because of continued discomfort despite conservative therapies and occasional âclickingâ of the wrist, she is taken to the operating room for diagnostic arthroscopy. At the time, fraying of the membranous portion of the scapholunate (SL) ligament is seen, with mild incongruity from the midcarpal joint. The surgeon is unable to pass a 1-mm probe through the defect. This is most consistent with:
1) Geissler Grade I SL tear
3) Geissler Grade III tear
2) Geissler Grade II SL tear
5) Scapholunate advanced collapse (SLAC) wrist
4) Geissler Grade IV tear
Arthroscopy has become the gold standard for the diagnosis of ligamentous injuries to the wrist. A classification scheme has been proposed by Geissler and colleagues, based on both radiocarpal and midcarpal arthroscopic findings (Table).
Table. Arthroscopic Classification of Interosseous Ligament Injury1 1 Grade Findings
-
Attenuation of the interosseous ligament with no radiocarpal or midcarpal step-off
-
Incongruence of the scapholunate interval seen from the midcarpal joint
-
Complete separation of scaphoid and lunate visualized from both spaces; a 1-mm probe can be passed between the two bones
-
Ability to pass 2.7-mm arthroscope between the scapholunate interval Correct Answer: Geissler Grade II SL tear
1592. (3636) Q4-6524:
A 33-year-old woman with a history of a traumatic fall onto her wrist and tenderness over the scapholunate (SL) interval presents to the clinic. Radiographs are normal, and magnetic resonance imaging reveals a partial tear of the SL ligament. The remaining wrist ligaments are normal. If conservative therapy is attempted, then it should consist of:
1) Short arm casting for 2 months
3) Splinting and flexor carpi radialis training
2) Activity modification and wrist extension stretching
5) Corticosteroid injection into the midcarpal space
4) Splinting and flexor carpi ulnaris training
Conservative management includes a period of splinting and activity modification, followed by proprioception training of the flexor carpi radialis to act as a dynamic scaphoid stabilizer.Correct Answer: Splinting and flexor carpi radialis training
1) Should be treated after 2 years of age
3) Usually presents in association with other systemic abnormalities
2) Usually presents bilaterally
5) Occurs in approximately 1 out of every 5,000 births
4) Often presents with a hypoplastic radial thumb and a dominant-appearing ulnar thumb
Experts recommend treating congenital thumb duplication before the age of 6 months, when the potential for growth and remodeling is greatest. The condition usually presents unilaterally, lacking association with other systemic abnormalities, and usually presents with a hypoplastic radial duplicate and dominant ulnar duplicate.Correct Answer: Often presents with a hypoplastic radial thumb and a dominant-appearing ulnar thumb
1594. (3663) Q4-7440:
Complete bifurcation of two distal phalanges articulating with a wide epiphysis of a single proximal phalanx is classified as:
1) Wassel II / IP
3) Wassel IV / MCP
2) Wassel III / IP
5) Wassel I/ Distal
4) Wassel IV / IP
Wassel II (also categorized as IP in the universal classification system) occurs when the duplication begins at the interphalangeal joint of the thumb, resulting in complete bifurcation of two distal phalanges that articulate proximally with a single proximal phalanx.Correct Answer: Wassel II / IP
1595. (3664) Q4-7441:
One of the more common complications of congenital thumb duplication reconstruction is:
1) Weakness of resulting digit
3) Ulnar deviation at metacarpophalangeal joint and radial deviation at interphalangeal joint
2) Paresthesias in resulting digit
5) Nail splitting
4) Nonhealing wound
A Z-deformity, with ulnar deviation at the MCP joint and radial deviation at the IP joint, is one of the most common complications after reconstruction. Weakness, paresthesias, and wound complications are uncommon possible complications.Correct Answer: Ulnar deviation at metacarpophalangeal joint and radial deviation at interphalangeal joint
1596. (3850) Q4-7633:
Ultrasound therapy delivers superficial heat to the tissue and has a penetration depth of 5 mm.
-
True
-
False
Ultrasound is considered a deep heat modality and does not heat the superficial tissues. Correct Answer: False
-
Increasing capsular extensibility
-
-
Increasing pain threshold
-
Decreasing scar
-
-
-
Increasing ligament stretch ability
-
Reversing Dupuytrenâs contracture
Thermal uses of ultrasound include increasing pain threshold, decreasing scar, and improving extensibility of the ligaments and joint capsule. Ultrasound has not been shown to have an effect in Dupuytrenâs contracture.
Correct Answer: Reversing Dupuytrenâs contracture
1598. (3852) Q4-7635:
Phonopheresis is:
1) Delivery of heat to the tissues with a special oval-shaped attachment
3) Delivery of medicine through the skin using ultrasound
2) Aspiration of blood with concentration of platelets for re-injection
5) Delivery of substimulus auditory waves to the tissue
4) Using ultrasound in a rapid, deep massage-type application
Phonopheresis is delivery of medicine through the skin using ultrasound. Although there is some question as to whether the medications are more effectively absorbed or delivered with ultrasound use, this is a described modality.
Correct Answer: Delivery of medicine through the skin using ultrasound
1599. (3853) Q4-7636:
Iontophoresis delivers medications such as analgesics or steroids through the skin using an electrical charge.
-
True
-
False
Iontophoresis uses a direct or galvanic current to âdriveâ medications transdermally and is used for scar management and pain control.
Correct Answer: True
1600. (3854) Q4-7637:
Iontophoresis has been effectively used in all of the following EXCEPT:
-
Carpal tunnel syndrome
-
-
Shoulder/rotator cuff tendinitis
-
Wrist arthritis
-
-
-
Medial epicondylitis
-
Lateral epicondylitis
Iontophoresis is effective in soft tissue conditions such as rotator cuff bursitis and lateral epicondylitis. Correct Answer: Wrist arthritis
-
Endoneurium
3) Perineurium
2) Hyponeurium
5) Epineurium
4) Mesoneurium
The structures surrounding the axons and Schwann cells include the endoneurium, perineurium, and epineurium. The mesoneurium is an adventitial layer in addition.
Correct Answer: Hyponeurium
1602. (3873) Q4-7656:
The Seddon grades of nerve injury include all of the following EXCEPT:
1) Neuropraxia
3) Neurotmesis
2) Axonotmesis
-
Schwann cell disruption
Neuropraxic injuries are stretch injuries to the nerve. Axonotmetic injury involves a more severe injury with loss of continuity of axons, and connective tissue elements remain intact. Neurotmesis is a complete nerve discontinuity. Schwann cell disruption occurs in neurotmesis but is not among the grades of Seddon nerve injury.
Correct Answer: Schwann cell disruption
1603. (3874) Q4-7657:
Younger age is associated with worse outcomes with nerve repair.
-
True
-
False
Younger age is associated with improved functional outcome after nerve repair, particularly regarding sensory recovery. Correct Answer: False
1604. (3875) Q4-7658:
Optimum conditions for nerve healing after direct repair include:
-
Gapping at suture repair site
-
-
Early motion of extremity
-
Tension-free repair
-
-
-
lacing sutures through the endoneurium to increase repair strength
-
Preservation of all tissue whether devitalized or viable
Tension-free repair is the optimal technique to improve the potential for nerve recovery. Gapping, failure to splint to prevent tension on the nerve with motion, and failure to excise scarred or devitalized nerve tissue are impairments to successful nerve repairs. Suture repairs through the deep nerve segments can damage the axons. Sutures should be placed through the epineurium or, in a grouped fascicular repair, through the perineurium around the fascicles.
Correct Answer: Tension-free repair
-
True
-
False
Missile wounds can cause a blast stretch injury to peripheral nerves and may recover with observation. Correct Answer: True
1606. (3887) Q4-7766:
The anatomic location of the pathologic lesion of lateral epicondylitis is the:
-
Extensor carpi radialis longus (ECRL)
-
-
Extensor digitorum longus (EDL)
-
Extensor carpi radialis brevis (ECRB)
-
-
-
Annular ligament
-
Extensor digitorum communis (EDC)
While the ECRL and EDL can sometimes be involved, the primary location of most cases of tennis elbow show characteristic changes at the origin of the ECRB.
Correct Answer: Extensor carpi radialis brevis (ECRB)
1607. (3888) Q4-7767:
Which of the following injectable substances have shown benefit in the treatment of lateral epicondylitis:
-
Corticosteroids
3) Botulinum toxin
2) Autologous blood
5) All of the above
4) None of the above
All of these substances have been shown to have efficacy in the treatment of tennis elbow. However, placebo saline injections have also been proven to add some benefit compared to no treatment.
Correct Answer: All of the above
1608. (4068) Q4-7768:
The nerve most at risk during arthroscopic debridement of lateral epicondylitis is the:
1) Ulnar nerve
3) Anterior interosseous nerve
2) Median nerve
5) Musculocutaneous nerve
4) Posterior interosseous nerve
While the ulnar nerve is most at risk during elbow arthroscopy in general, debridement of the lateral capsule posterior to the midpoint of the radiocapitellar joint places the posterior interosseous branch of the radial nerve at risk.
Correct Answer: Posterior interosseous nerve
all of the following, except:
1) Synovial plica
3) Synovitis
2) Loose body
5) Chondral lesion
4) Medial epicondylitis
While all of the other answers are intra-articular lesions that have been reported in elbow arthroscopies, medial epicondylitis is an extra-articular condition and must be addressed in an open fashion given the proximity of the ulnar nerve.
Correct Answer: Medial epicondylitis
1610. (3890) Q4-7770:
Arthroscopic resection/debridement posterior to the midline of the radio-capitellar joint can result in damage to ligament, resulting in instability.
1) Lateral collateral; valgus
3) Ulnar collateral; posterolateral rotatory
2) Ulnar collateral; valgus
5) Lateral collateral; posterolateral rotatory
4) Annular; posterolateral rotatory
Resection posterior to the midpoint of the radiocapitellar joint can result in damage to the lateral collateral ligament and subsequent development of posterolateral rotator instability.
Correct Answer: Lateral collateral; posterolateral rotatory
1611. (3911) Q4-7791:
The muscle that flexes the interphalangeal joint of the thumb is innervated by which roots of the brachial plexus:
1) C 5, C 6
3) C 6, C 7, C 8
2) C 5, C 6, C 7
5) C 7, C 8, T1
4) C 5, C 6, C 7, C 8, T1
The interphalangeal joint of the thumb is flexed secondary to actions of the flexor pollicis longus (FPL). The FPL is innervated by the anterior interosseous nerve, which is the longest branch of the median nerve. The median nerve is formed by the lateral (roots C 5, C 6, and C 7) and medial (roots C 8 and T1) cords of the brachial plexus.
Correct Answer: C 5, C 6, C 7, C 8, T1
1612. (3912) Q4-7792:
The anterior interosseous nerve (AIN) originates from the median nerve at what distance from the medial epicondyle:
1) 6 cm distal
3) 6 cm proximal
2) 10 cm proximal
5) At the medial epicondyle
4) 10 cm distal
The AIN, the largest branch of the median nerve, originates 5 cm to 8 cm distal to the medial epicondyle from the posteroradial aspect of the median nerve just distal to the proximal border of the superficial head of the pronator teres.
Correct Answer: 6 cm distal
A 30-year-old right-hand dominant woman presents to the emergency department with a 6-week history of difficulty writing and pain after playing tennis. She also reports a recent inability to abduct and adduct her fingers. What is the mechanism of her symptoms:
1) Writerâs cramp or focal dystonia
3) Martin-Gruber interconnection
2) Riche-Cannieu anastamosis
5) Carpal tunnel syndrome
4) Ulnar neuropathy
Approximately 17% of the population has a Martin-Gruber interconnection, and 50% of these patients may show additional denervation of normally ulnar nerve-innervated intrinsic muscles. The Martin-Gruber anomaly is a motor neural connection between the anterior interosseous nerve and ulnar nerves that is located adjacent to the ulnar artery in the proximal forearm.
|
Correct Answer: Martin-Gruber interconnection 1614. (3914) Q4-7794:
What is the innervation of the indicated muscle in the image (Slide):
1) Median nerve
3) Radial nerve
2) Anterior interosseous nerve
5) Posterior interosseous nerve
4) Ulnar nerve
The arrow in the photograph (Slide) is pointing to the pronator teres â one of the most common sites for compression of the anterior interosseous nerve. The pronator teres is innervated by the median nerve.
Correct Answer: Median nerve
1615. (3915) Q4-7795:
Sites of potential compression of the median nerve include all of the following except:
1) Pronator teres
3) Pisohamate ligament
2) Transverse carpal ligament
5) Mass in carpal canal (e.g., lipoma)
4) Supracondylar process
Around the elbow, the median nerve may be compressed by the pronator teres (causing either anterior interosseous nerve syndrome or pronator syndrome) or the ligament of Struthers originating from a supracondylar process (causing pronator syndrome). In the wrist, the median nerve may be compressed by the transverse carpal ligament or a mass within the carpal canal. The ulnar nerve, not the median nerve, can be compressed by the pisohamate ligament.
Correct Answer: Pisohamate ligament
What position of the wrist most commonly produces scaphoid fractures:
1) Wrist flexion and radial deviation
3) Wrist flexion and ulnar deviation
2) Wrist extension and radial deviation
5) Clenched fist and wrist flexion
4) Wrist extension and ulnar deviation
Frykman performed biomechanical studies to evaluate the wrist position in falls that produce scaphoid fractures. The results showed that wrist extension greater than 90° and radial deviation consistently resulted in fracture of the scaphoid.
Correct Answer: Wrist extension and radial deviation
1617. (3939) Q4-8127:
Which of the following blood vessels supplies the majority of the scaphoid:
1) Superficial palmar branch of the radial artery (volar)
3) Dorsal carpal branch of the radial artery (dorsal)
2) Radial artery
5) 3,4 intracompartmental supra-retinacular artery (3,4-ICSRA)
4) Ulnar artery
Gelberman and Menon used injection studies to demonstrate that the majority of scaphoid blood flow stems from branches of the radial artery entering the scaphoid at the distal pole. Of these, the branch entering the dorsal ridge supplies 70% to 80% of the intraosseous vascularity of the scaphoid bone. The proximal pole is completely dependent on the intraosseous blood supply and is vulnerable to avascular necrosis when fracture disrupts this vascular source.
Correct Answer: Dorsal carpal branch of the radial artery (dorsal)
1618. (3940) Q4-8128:
During a posterior (dorsal) approach to percutaneous screw fixation for a scaphoid fracture, many structures are close to the guidewire insertion location and are at risk for injury. Which of the following structures is the closest to the guidewire insertion location according to a recent cadaveric study:
1) Posterior interosseous nerve
3) Extensor indicis proprius
2) Extensor digitorum communis to the index
5) Extensor digitorum communis to the index and posterior interosseous nerve
4) Extensor carpi radialis brevis
Adamany and colleagues performed a cadaveric study to evaluate the dorsal structures at risk with truly percutaneous headless screw placement for scaphoid fractures. They noted that the posterior interosseous nerve and the extensor digitorum communis to the index finger were an average of 2.2 mm from the guidewire and therefore at greatest risk from this approach.
Correct Answer: Extensor digitorum communis to the index and posterior interosseous nerve
Which of the following is a concerning risk factor for a dorsal open approach to the scaphoid:
1) Damage to tenous blood supply of the scaphoid
3) Damage to scapho-trapezial-trapezoid joint during the approach
2) Difficulty of central screw placement
5) njury to the higher rate of infection
4) Damage to the lunatotriquetral (LT) ligament
The dorsal approach is advantageous in obtaining central screw placement. The scapho-trapezial-trapezoid joint is at risk during a volar approach, not a dorsal approach, and infection has not been shown to occur more frequently in one approach over the other. The LT ligament is not seen in either approach. The risk of the open dorsal approach is compromise of the main blood supply to the scaphoid, entering through the dorsal ridge.
Correct Answer: Damage to tenous blood supply of the scaphoid
1620. (3942) Q4-8130:
A volar approach to the scaphoid is ideal in which of the following fractures:
1) Proximal pole fractures
3) Comminuted scaphoid fractures
2) Distal pole fractures
5) Scaphoid fracture with associated scapholunate ligament tear
4) Avascular necrosis of the scaphoid proximal pole
The volar approach to the scaphoid is optimal in distal pole fractures because it allows direct visualization of the fracture line and exact reduction and fixation. A volar approach is not recommended for fractures or avascular necrosis of the proximal pole, where dorsal screw placement is best. Scapholunate ligament tears are generally repaired from a dorsal approach because the ligament is stoutest dorsally.
Correct Answer: Distal pole fractures
1621. (3944) Q4-8195:
Which finger is most commonly involved in a flexor digitorum profundus (FDP) avulsion injury:
1) Index
3) Ring
2) Middle
5) Thumb
4) Small
An FDP avulsion is caused by forceful extension of the distal interphalangeal (DIP) joint, occurring at the same time as a maximum contraction of the FDP tendon. Jersey finger is often seen in athletes, especially football or rugby players, who commonly get their fingers caught in an opposing playerâs jersey, thus the name. In most cases, this injury affects the ring finger.Correct Answer: Ring
1622. (3945) Q4-8196:
Which type of flexor digitorum profundus (FDP) avulsion is considered the most severe:
1) Type I
3) Type III
2) Type II
5) Type V
4) Type IV
Type I is the most threatening scenario because the FDP tendon retracts into the palm, and vincular and diffusional blood supply is lost. The sheath may be noncompliant after a few days and may not allow passage of the FDP tendon through the sheath in an attempt to repair the stump to its insertion. Additionally, proximal muscle contracture prevents tendon stump advancement.Correct Answer: Type I
When performing pollicization to correct a hypoplastic thumb, the surgeon should rotate the index finger:
1) 120º
3) 150º
2) 135º
5) 180º
4) 165º
After the index finger is rotated 150º, the index finger will be in the ideal location as it mimics the position of where the thumb would have naturally been. This position allows for the greatest amount of grip and pinch strength possible.Correct Answer: 150º
1624. (3947) Q4-8198:
For which types of thumb hypoplasia is pollicization the best option:
1) Type I
3) Type IIIA
2) Type II
5) Type I, type II, and type IIIA
4) Type IIIB
Reconstruction is possible and is therefore the best option for thumb hypoplasia in patients with type I, type II, and type IIIA. These three types of hypoplasia can be corrected because the thumb still has most of the bones and muscles intact. Corrective surgery is necessary to correct weak muscles or a tight web space between the thumb and index finger. When a type IIIB exists, reconstruction is not possible and pollicization must be performed.Correct Answer: Type I, type II, and type IIIA
1625. (3948) Q4-8199:
What is the most critical step in pollicization to create a normal-looking thumb:
1) Creating skin incisions with skin flaps that will allow a natural first web space
3) Shortening of the index finger metacarpal
2) Shortening of tendons
5) All of the above
4) Creating a hyperextended joint when stabilizing the metacarpophalangeal joint to the carpus
It is necessary to create a hand with a natural first web space, shorter tendons that allow for natural movement, a shorter metacarpal that ensures the finger will not grow to an unnatural length, and a hyperextended joint to create the most natural-looking hand possible.Correct Answer: All of the above
1626. (3949) Q4-8200:
All of the following may be present in a child with type IIIA hypoplasia except:
1) Metacarpophalangeal (MP) joint laxity
3) Lack of extensor pollicis longus
2) Web space contracture
5) Thenar muscle atrophy
4) Unstable carpometacarpal (CMC) joint
Type IIIA hypoplasia includes web space narrowing, thenar atrophy, MP joint laxity, and extrinsic tendon abnormalities. Type IIIA is distinguished from a type IIIB by the fact that a stable CMC joint exists. Because a stable CMC joint exists, a reconstruction is the treatment of choice. When the CMC joint is unstable, as in type IIIB hypoplasia, a pollicization is necessary to restore thumb stability.Correct Answer: Unstable carpometacarpal (CMC) joint
The main 3-4 viewing portal for wrist arthroscopy lies in between which two tendons:
1) Extensor pollicis longus (EPL) and extensor carpi radialis brevis (ECRB)
3) Abductor pollicis longus (APL) and extensor carpi radialis longus (ECRL)
2) Extensor digitorum communis (EDC) and extensor digiti minimi (EDM)
5) Extensor pollicis brevis (EPB) and APL
4) EPL and EDC
The 3-4 portal is the main viewing portal and is located between the third and fourth compartment. This portal is bordered by the extensor digitorum communis (EDC) to the index finger, and the extensor pollicis longus (EPL) can be palpated in the âsoft spotâ 1 cm distal to Listers tubercle. This portal is usually the first portal to be made during wrist arthroscopy.Correct Answer: EPL and EDC
1628. (3951) Q4-8202:
Which of the following ligaments acts as a neurovascular conduit:
-
Long radiolunate
3) Radioscapholunate
2) Radioscaphocapitate
5) Ulnotriquetral
-
Short radiolunate
The radioscapholunate ligament, otherwise known as the ligament of Testut, lacks structural intergrity and acts as a neurovascular conduit. This ligament is visible on the volar side of the wrist from the 3-4 portal in between the long radiolunate and short radiolunate ligaments.Correct Answer: Radioscapholunate
1629. (3952) Q4-8203:
Complications after wrist arthroscopy occur in what percentage of patients:
1) 5%
3) 15%
2) 10%
-
25%
4) 20%
The complication rate after routine wrist arthroscopy is between 2% and 5%.Correct Answer: 5%
1630. (3953) Q4-8204:
Complications after wrist arthroscopy occur in what percentage of patients:
1) 5%
3) 15%
-
-
10%
5) 25%
4) 20%
The complication rate after routine wrist arthroscopy is between 2% and 5%.Correct Answer: 5%
The fracture fragment in Bennettâs fracture is located in which of the following areas of the hand:
-
Radiopalmar trapezium
-
-
Ulnopalmar trapezium
-
Dorsal thumb metacarpal base
5) Radiopalmer lunate
4) Ulnopalmar thumb metacarpal base
As an axial load is placed on the thumb tip, it drives the thumb metacarpal (MC) base in a dorsal-radial direction. As the thumb MC base moves dorsoradially, a fracture is created in the volar, ulnar quadrant of the thumb MC base. Gedda and Moberg describe this as a ligament fracture avulsion. The volar, ulnar quadrant piece usually remains stationary, perhaps migrating a small amount distal the thumb metacarpal base moves dorsoradially, creating a fracture in the volar, ulnar quadrant of the trapezium.Correct Answer: Ulnopalmar thumb metacarpal base
1632. (3955) Q4-8206:
Which of the following two main soft tissue forces are disrupted by Bennettâs fracture subluxation:
1) Volar beak (anterior oblique) ligament and extensor pollicis longus
3) Posterior oblique ligament and abductor pollicis brevis
2) Volar beak (anterior oblique) ligament and abductor pollicis longus
5) Dorsal radial ligament and abductor pollicis brevis
4) Dorsal radial ligament and flexor pollicis brevis
The volar, ulnar quadrant piece usually remains stationary due to the volar beak ligament. The thumb metacarpal base tends to sublux dorsoradially due to unopposed pull of the abductor pollicis longus and adductor pollicis. The intact volar beak ligament is usually the counterforce the to these two muscles in the static situation. The extensor pollis longus, flexor pollicis brevis, and abductor pollis longus do not have significant involvement in the Bennettâs fracture subluxation. Although the dorsal radial ligament is important for carpometacarpal stability, it is not the ligament attached to the fractures fragment.Correct Answer: Volar beak (anterior oblique) ligament and abductor pollicis longus
1633. (3956) Q4-8207:
The greatest amount of step-off that is well-tolerated in a Bennettâs fracture is:
1) 0 mm
3) 2 mm to 3 mm
2) 1 mm to 2 mm
5) 4 mm to 5 mm
4) 3 mm to 4 mm
Studies by Livesley, Kjaer-Petersen, and others have shown that patients with fractures with more than a 1-mm step-off after reduction were more likely to develop arthritis at the thumb carpometacarpal joint. Although some studies have not shown functional outcome correlating with the presence of arthritis, Oosterbos and De Boer found that all their patients with fair and poor overall results had nonanatomic reductions. Although a cadaveric study by Cullen has shown that a 2-mm step-off may be acceptable, this contrasts with the clinical evidence currently available.Correct Answer: 1 mm to 2 mm
1634. (3957) Q4-8208:
When fracture step-off is greater than the accepted limits, which of the following complications is the most common:
1) Arthritis
3) Decreased range of motion
2) Pain
5) All of the above
4) Decreased pinch strength
Studies by Livesley, Kjaer-Petersen, and others have shown that patients with fractures with more than a 1-mm step-off after reduction were more likely to develop arthritis at the thumb carpometacarpal joint. Pain, decreased range of motion, and decreased pinch strength also correlated with these poor outcomes.Correct Answer: All of the above
Clinically, what is the upper limit of acceptable fracture angulation for a fifth metacarpal neck fracture:
1) 20°
3) 50°
2) 40°
5) 80°
4) 70°
Although this is controversial, conservatively treated patients with angulations less than 70° fared well in two prospective studies. Fourteen percent of patients will have a cosmetic deformity, but operatively treated patients exhibited extensor lag and increased rehabilitation times.Correct Answer: 70°
1636. (3959) Q4-8210:
In cadaveric models, when does the biomechanics of fifth finger flexion consistently change in relationship to metacarpal neck fracture angulation:
1) 10°
3) 50°
2) 30°
5) 80°
4) 70°
Thirty degrees of angulation is the maximum deformity for acceptable fifth finger grip strength. Ali et al showed that fracture angulation of 30° results in a significant decrease in the distance between the origin and the insertion of the flexor digiti minimi (FDM). This shortening creates more âslackâ in the FDM muscle and more excursion is wasted as muscle shortening prior to the initiation of metacarpophalangeal (MP) flexion.Correct Answer: 30°
1637. (3960) Q4-8211:
Up to how much angulation can be tolerated in the small finger metacarpal shaft fracture:
1) 0° to 10°
3) 21° to 30°
2) 11° to 20°
5) 41° to 50°
4) 31° to 40°
The small finger carpometacarpal joint is mobile, which allows the small finger metacarpal to tolerate deformity better than the fixed index and middle finger rays. Thus, 41° to 50° of angulation can be accommodated by the mobile carpometacarpal joint.Correct Answer: 41° to 50°
1638. (3961) Q4-8212:
If a metacarpal shaft fracture shortens 4 mm, what will the theoretical amount of extensor lag be at the metacarpophalangeal joint:
1) 0°
3) 7°
2) 5°
5) 20°
4) 14°
For each 2 mm of shortening, a 7° extensor lag exists. Thus, with 4 mm of shortening, there will be a 14° extensor lag at the metacarpophalangeal joint.Correct Answer: 14°
In a short oblique metacarpal shaft fracture without comminution or bone loss, what is usual amount of maximal shortening that will occur:
1) 1 mm
3) 5 mm
2) 3 mm
5) 9 mm
4) 7 mm
In a cadaveric study, shortening beyond 5 mm was prevented by the tethering effect of the transverse metacarpal ligaments and adjacent metacarpals.Correct Answer: 5 mm
1640. (3962) Q4-8214:
Which of the following statements is true regarding metacarpophalangeal joint anatomy:
1) The collateral ligaments are lax in flexion.
3) Joint stability is maximal in flexion.
2) The joint volume is highest in flexion.
5) The collateral ligaments originate volar to the axis of flexion.
4) The metacarpal head is spherical.
The collateral ligaments are lax in extension and tight in flexion. The joint volume is highest in extension. The metacarpal head is cam-shaped. The collateral ligaments originate dorsal to the axis of flexion. Due to the tightening of the collateral ligaments over the cam-shaped metacarpal head in flexion, joint stability is maximized.Correct Answer: Joint stability is maximal in flexion.
1641. (3963) Q4-8215:
Which of the following fracture patterns and mechanisms is incorrectly paired:
1) Transverse fracture-direct blow
3) Comminuted fractures with a butterfly fragment-axial compression and bending
2) Transverse fracture-axial load on an extended metacarpophalangeal joint
5) Oblique-torsion and axial load
4) Spiral fracture-torsion
Biomechanically and clinically, fracture patterns are often associated with certain types of force. Transverse fractures occur with a direct blow, comminuted fractures occur with axial compression and bending, spiral fractures occur in torsion, and oblique fractures occur with torsion and axial load.Correct Answer: Transverse fracture-axial load on an extended metacarpophalangeal joint
1642. (3964) Q4-8216:
Giant cell tumor of tendon sheath commonly occurs in which of the following age groups:
1) Infants (age 0-1 year)
3) Age 10-20 years
2) Age 1-10 years
5) Age 60-70 years
4) Age 30-40 years
Giant cell tumor of tendon sheath is most commonly found in patients in the fourth through sixth decades; therefore, age 30-40 years is the most appropriate answer choice.Correct Answer: Age 30-40 years
Which of the following clinical features is common in giant cell tumor of tendon sheath:
1) Transillumination
3) Fluctuates in size
2) Erythematous
5) Painless
4) Presents with rapid change in size
Giant cell tumor of tendon sheath is painless. Giant cell tumor of tendon sheath does not transilluminate, as ganglion cyst does. No overlying skin color changes occur. Giant cell tumor of tendon sheath only increases in size and does not fluctuate like a ganglion cyst; it does not present with a rapid increase in size.Correct Answer: Painless
1644. (3966) Q4-8218:
After plain radiographs of giant cell tumor of tendon sheath are obtained, the following imaging study should be obtained:
1) Computed tomography scan
3) Magnetic resonance image
2) Ultrasound
5) Bone scan
4) Angiogram
Magnetic resonance imaging provides anatomic detail of the soft tissue mass, helps generate a differential diagnosis, and determines if the mass is unifocal or multifocal and where it originates. Giant cell tumor of tendon sheath is a soft-tissue tumor. Computed tomography is best for bone-based tumors. Ultrasound helps localize lesions but does not provide anatomic detail to help determine the type of mass. Although angiograms are useful for vascular tumors such as renal cell carcinoma or arteriovenous malformations, they are not necessary in the evaluation of a soft tissue mass in the hand with features suggestive of giant cell tumor of tendon sheath. A bone scan is useful when malignant bone tumors are suspected rather than benign soft tissue masses.Correct Answer: Magnetic resonance image
1645. (3967) Q4-8219:
Which of the following cell types is not typically found in giant cell tumors of tendon sheath:
1) Multinucleated giant cells
3) Monocytes
2) Histiocytes
5) Fibroblasts
4) Polymorphonuclear lymphocytes
Multinucleated giant cells, histiocytes, monocytes, and fibroblasts are commonly found in pathologic giant cell tumor of tendon sheath specimens. Polymorphonuclear lymphocytes are typically associated with bacterial infections.Correct Answer: Polymorphonuclear lymphocytes
|
A 25-year-old, right-hand-dominant male truck driver presents to the emergency department (Slide 1, Slide 2). The tip of his left ring finger was amputated in a bicycle accident 2 weeks prior. The amputated piece was âsewn back onâ in the emergency department immediately after the accident, but âturned blackâ over the next week. There is no evidence of infection. He states that the appearance of his finger is embarrassing, and he would like it taken care of as soon as possible. Which of the following procedures is the most appropriate:
1) Local debridement, allow to heal by secondary intention
3) Kutler V-Y advancement flap closure
2) Atasoy-Kleinert V-Y advancement flap closure
5) Split-thickness hypothenar skin graft
4) Moberg flap closure
The Atasoy-Kleinert V-Y advancement flap is the best option for transversely oriented fingertip amputations/defects and also for defects with more dorsal than volar tissue loss. The apex of the V is positioned at, or just distal, to the distal interphalangeal joint crease on the volar side of the digit. After incising the V marking, the flap is advanced distally to cover the defect, and the incisions are closed in a Y pattern.
Local, or chemical, debridement and allowing the resulting defect to heal by secondary intention are a viable option, but the patient stated that he would prefer an aggressive treatment protocol because the appearance of his fingertip is so embarrassing.
The Kutler (lateral) V-Y advancement flap is typically used to cover tip defects that demonstrate more volar than dorsal tissue loss. The procedure involves creating V-Y advancement flaps laterally on either side of the affected digit and advancing them toward each other in the midline thereby covering the defect.
The Moberg flap is typically used for reconstruction of thumb amputations. This procedure involves the creation of volar tissue flap that includes the neurovascular bundles on either side of the digit. Its use is cautioned in very distal amputations because excess stretch on the vascular pedicles may lead to necrosis at the tip of the flap. Its use is also cautioned in the fingers because of the difference in orientation of the blood supply compared to the thumb.
A full-thickness, rather than a split-thickness, skin graft is a viable option to manage this patient. Skin grafts for hand reconstruction should be harvested with the âlike replaces likeâ principle in mind, especially when reconstructing the volar skin. Volar hand skin is much thicker and of unique quality when compared with the rest of the body, and therefore, the most appropriate place to harvest a skin graft is the volar surface of the hand.
Correct Answer: Atasoy-Kleinert V-Y advancement flap closure
A 52-year-old, right-hand-dominant watchmaker arrives at the emergency department 30 minutes after the volar soft tissue of his right thumb and index finger was avulsed while using a bandsaw. Physical examination shows 2 cm 3 2 cm wounds involving the distal phalanx of each affected digit. No exposed tendon or bone is present, and no involvement of the joints is noted. The patient requests a treatment option that will retain the most sensation so he can effectively continue in his occupation. Which of the following options is the most appropriate management of this patientâs wounds:
1) Coverage with cross-finger flaps
3) Split-thickness skin grafting
2) Healing by secondary intention
5) Radial free forearm flap
4) Full-thickness skin grafting
Local flaps such as cross finger flaps are good options but require at least two surgeries (inset then division) and can often result in stiffness secondary to the requisite period of immobilization. In addition, local flaps have lesser return of sensibility than the other techniques listed.
Return of tactile sensibility is excellent after healing by secondary intention, but dressing changes for wounds that measure 2 cm 3 2 cm would take months to completely heal.
Skin grafting is the next available option with acceptable sensory return. It can be performed during local anesthesia, requires only one operation, and allows for early motion thereby avoiding stiffness. Studies have shown that full-thickness skin grafts recover sensation better than split-thickness skin grafts.
A radial forearm flap will be excessively bulky, has unacceptable donor site morbidity in this situation, and results in inadequate sensory recovery.
Correct Answer: Full-thickness skin grafting
1648. (3970) Q4-8222:
Which of the following is not considered a part of the triangular fibrocartilage complex:
1) Ulnolunate ligament
3) Dorsal radioulnar ligament
2) Palmar radioulnar ligament
5) Ulnotriquetral ligament
4) Radiolunate ligament
The triangular fibrocartilage complex is made up of the dorsal and palmar radioulnar ligaments, the meniscal homologue, the articular disk, the ulnolunate, and the ulnotriquetral ligaments. The radiolunate ligament is not part of the complex.Correct Answer: Radiolunate ligament
1649. (3971) Q4-8223:
Which of the following arterial branches does not supply the peripheral 25% of the triangular fibrocartilage complex:
1) Dorsal branch of the anterior interosseous artery
3) Dorsal branch of the radial artery
2) Palmar branch of the anterior interosseous artery
5) Palmar branch of the ulnar artery
4) Dorsal branch of the ulnar artery
The triangular fibrocartilage complex is supplied by both branches of the anterior interosseous artery and the ulnar artery; it is not supplied by the dorsal branch of the radial artery.Correct Answer: Dorsal branch of the radial artery
Which of the following statements is true:
1) In a wrist with neutral ulnar variance, 20% of the axial load is transmitted across the ulna.
3) In a wrist with 2.5 mm ulnar negative variance, 60% of the axial load is transmitted across the ulna.
2) In a wrist with 2.5 mm ulnar negative variance, 20% of the axial load is transmitted across the ulna.
5) In a wrist with 2.5 ulnar positive variance, 20% of the axial load is transmitted across the ulna.
4) In a wrist with 2.5 mm ulnar positive variance, 600% of the axial load is transmitted across the ulna.
Cadaveric studies have been performed to determine the amount of load across the wrist with various relationships between the radius and ulna lengths. In wrists with neutral ulnar variance (in which the radius and ulna are equal in length), 20% of the load is transmitted across the ulna and 80% is transmitted across the radius. In wrists with negative ulnar variance (in which the ulnar is shorter than the radius), more load is transmitted across the radius and less is transmitted across the ulna. The opposite is true with positive ulnar variance (the ulna is longer than the radius).Correct Answer: In a wrist with neutral ulnar variance, 20% of the axial load is transmitted across the ulna.
1651. (3973) Q4-8225:
Which of the following parameters is not a determinant of the Palmer classification of triangular fibrocartilaginous complex injuries:
1) Location of the lesion
3) Presence of lunatotriquetral ligament injury
2) Presence of ulnar head chondromalacia
5) Presence of ulnocarpal arthritis
4) Size of the lesion
The Palmer classification divides triangular fibrocartilage complex lesions into traumatic and degenerative. Traumatic subclassifications are based on the location of the ligament tear. In the degenerative tear, subclassifications are based on the degree of injury to the triangular fibrocartilage complex and associated chondral and ligamentous injury. Thus, the size of the lesion is not a parameter in the Palmer classification.Correct Answer: Size of the lesion
1652. (3974) Q4-8226:
What are the components of a Galeazzi fracture-dislocation:
1) Triangular fibrocartilage complex (TFCC) tear, interosseous membrane tear, and radial shaft fracture
3) Interosseous membrane tear and radial shaft fracture
2) Ulnar shaft fracture, interosseous membrane tear, and TFCC tear
5) Wrist radial collateral ligament tear, dorsal intercarpal ligament tear, and ulnar shaft fracture
4) TFCC tear and radial shaft fracture
Only one in vitro study examined the soft tissue constraints of the Galeazzi fracture-dislocation pattern. Moore and colleagues performed a radial osteotomy at the pronator teres insertion of nine cadaveric forearms and then sectioned the TFCC and the interosseous membrane in alternating orders. They found that all three structures (TFCC, interosseous membrane, and radial shaft) must be injured to create a radial shortening of more than 10 mm in relationship to the distal ulna.Correct Answer: Triangular fibrocartilage complex (TFCC) tear, interosseous membrane tear, and radial shaft fracture
1653. (3975) Q4-8227:
Who are the most common athletes to get medial epicondylitis of the elbow:
1) Tennis players
3) Swimmers
2) Golfers
5) Basketball players
4) Gymnasts
Although medial epicondylitis is called golferâs elbow, tennis players are more likely to have this condition. Medial epicondylitis can occur in any sport such as baseball pitching, javelin throwing, swimming, and gymnastics in which athletes place a significant valgus flexion force on their elbow.Correct Answer: Tennis players
1) A crush injury in a patient who smokes
3) An ischemia time of 24 hours
2) A sharp amputation of the thumb in a 15-year-old patient
5) A single digit amputation proximal to the flexor digitorum superficialis (FDS) insertion
4) The use of an amputated part that was immersed in warm water for transport
A sharp amputation, particularly of the thumb, is the best indication for replantation. Crushed digits, prolonged ischemia time, poor condition of the severed part, and single digit loss proximal to the FDS insertion on the middle phalanx are relative contraindications to replantation.
Correct Answer: A sharp amputation of the thumb in a 15-year-old patient
1655. (3993) Q4-8245:
An amputation through the wrist is an indication for attempted replantation.
-
True
-
False
An amputation through the wrist, palm, or forearm is an indication for attempted replantation. The caliber of the vessels and other structures provides a favorable environment for reconstruction.
Correct Answer: True
1656. (3994) Q4-8246:
Care of an amputated part prior to replantation includes:
-
Painting the amputated part with povidone-iodine
-
-
Placing the amputated part in a warm saline bath
-
Immersing the amputated part in water
5) Putting the amputated part next to the patientâs body to keep it warm
4) Wrapping the amputated part with saline-soaked gauze and placing it in a plastic bag on ice
The appropriate care of an amputated part includes wrapping it in saline-dampened gauze and placing it on ice in a watertight bag. These actions preserve the tissues and slow cellular death until replantation is attempted. The part should not be immersed, painted with povidone-iodine, or kept next to the body.
Correct Answer: Wrapping the amputated part with saline-soaked gauze and placing it in a plastic bag on ice
1657. (3995) Q4-8247:
Replants are monitored by:
1) Color
3) Doppler probes
2) Turgor
5) All of the above
4) Temperature measurements
Monitoring of replanted parts postoperatively is accomplished by clinical checks of color or turgor to indicate blood flow. Additional objective monitoring is performed by using Doppler probes to check flow or by measuring temperature differences between the replanted part as compared to other digits.
Correct Answer: All of the above
Complications post-replantation include:
1) Cold intolerance
3) Excessive laxity of the digit
2) Stiffness
5) All of the above
4) A and B only
Complications after replantation include cold intolerance, which may improve over time, and stiffness of the replanted digit, which is generally due to immobility, tendon adhesions, and joint contracture.
Correct Answer: A and B only
1659. (4003) Q4-8255:
Placing some tension on a flexor tendon repair increases the ultimate tensile strength of the repair.
-
True
-
False
Tension on the repair site of a flexor tendon laceration has been shown experimentally to increase the tensile strength of the repair.
Correct Answer: True
1660. (4004) Q4-8256:
Flexor tendon nutrition is derived from:
-
The vincula
-
-
Synovial diffusion
-
The pulleys
5) A and C only
4) All of the above
Flexor tendon nutrition in the uninjured state is derived via the vincula, which contain blood vessels for nutrition. Injured tendons obtain nutrition via diffusion of synovial fluid. The pulley system does not contribute to flexor tendon nutrition.
Correct Answer: A and C only
1661. (4005) Q4-8257:
Immobilization as a postoperative therapy for flexor tendon repair is recommended for:
1) Lacerations that involve both the flexor digitorum superficialis tendon and the flexor digitorum profundus tendon
3) Children
2) 2-strand tendon repairs
5) Flexor tendon laceration in the thumb
4) Associated pulley rupture
As children have difficulties in following the detailed flexor tendon rehabilitation program that is recommended for adults after flexor tendon repair, it is advisable to completely immobilize them to protect the repair and avoid inadvertent rupture.
Correct Answer: Children
Optimization of early active motion protocols for flexor tendon rehabilitation includes:
1) Multi-strand repair
3) Using passive flexion of the finger to gain flexibility
2) Splinting the digit in extension
5) Ultrasound as an adjunct
4) Therapist hyperextension of the finger
The use of 6- and 8-strand repair techniques allow the flexor tendon repair to withstand the force applied by early active motion protocols. The addition of epitendinous tendon repair also strengthens the repair.
Correct Answer: Multi-strand repair
1663. (4007) Q4-8259:
The splint for early active motion flexor tendon rehabilitation protocols includes:
1) A wrist flexion piece
3) A yoke over the affected finger
2) A dynamic extension outrigger
5) None of the above
4) A hinge at the wrist to allow a tenodesis effect
A hinge at the wrist, which provides a tenodesis effect, allows the patient to passively extend the wrist and flex the fingers in preparation for gentle muscle contraction of the fingers.
Correct Answer: A hinge at the wrist to allow a tenodesis effect
1664. (4010) Q4-8262:
A 13-year-old boy tears his anterior cruciate ligament (ACL) while playing flag football. What is the preferred graft material for his ACL reconstruction:
1) Bone-patellar tendon-bone autograft
3) Four-strand hamstring tendon autograft
2) Quadriceps tendon autograft
5) Achilles tendon allograft
4) Four-strand hamstring tendon allograft
Due to the patients age, autograft is the preferred option. Also, due to the patientâs age, his growth plates are open and the surgeon is prohibited from using a graft with a bone construct due to the possible damage to the growth plate.
Correct Answer: Four-strand hamstring tendon autograft
1665. (4021) Q4-8274:
What deformity can develop in a mistreated volar PIP joint dislocation?
1) Swan neck deformity
3) Boutonniere deformity
2) Extensor lag
5) Hyperextension deformity
-
Flexion contracture
With volar PIP joint dislocations, there is almost always a disruption of the central slip of the extensor tendon. Because the central slip is involved, the PIP joint will hold a flexed position, and the lateral bands will fall volar to the axis of rotation of the PIP joint. The lateral bands will then exacerbate the flexion at the PIP joint, and due to their pull on the terminal tendon at the insertion on the distal phalanx, the DIP joint will hyperextend. This results in a boutonniere deformity.Correct Answer: Boutonniere deformity
What percentage of the articular surface must be involved in a dorsal PIP joint fracture dislocation for disruption of the collateral ligaments to occur?
1) 20%
3) 60%
2) 40%
-
100%
-
-
80%
The percentage of articular surface involved differentiates a stable dorsal PIP joint fracture dislocation from an unstable one. It is believed that when the involved fracture fragment is less than 40% of the articular surface, the insertion site of the collateral ligaments is not disrupted and the joint is stable. If more than 40% of the articular surface is fractured, then the insertion of the collateral ligaments is involved and the joint will subsequently be unstable.Correct Answer: 40%
1667. (4023) Q4-8276:
Which structures are disrupted in a lateral PIP joint dislocation?
-
Volar plate by itself